4120 1350
Exam Time
-
NOTEPAD
Results
of 200 questions answered correctly

You have reached of 200 points, ( %)

Your time

Question 1 of 200

1.

You are a doctor on duty. A patient after a successful resuscitation (drowning) was delivered to an admission room. BP is 90/60 mm Hg, heart rate is 120/min., respiration rate is 26/min. The patient is unconscious,  pupils are moderately dilated,  general  clonic and  tonic convulsions are observed. Make the diagnosis:

Explanation

 

Notice that this patient has just been resuscitated;  postresuscitation disease is a specific pathophysiologic state of vital organ systems  immediately after an episode of ischemic anoxia ( insufficient supply of oxygen to the brain due to a drop in cerebral blood flow or arterial pressure).

2.

A  32-year-old  welder  complains  of weakness  and fever. His illness initially presented as tonsillitis one month  earlier.  On examination: BT- 38, 9o C , RR-  24/min., HR-100/min.,  BP-  100/70 mm  Hg,  hemorrhages on the legs, enlargement of the lymph nodes. CBC  shows  Hb-  70  g/l,  RBC-  2, 2 · 1012 /l, WBC-  3, 0 · 109 /l with 32%  of blasts,  1%  of eosinophiles, 3% of bands, 36% of segments, 20%  of lymphocytes,  and  8%  of monocytes, ESR-   47  mm/hour.   What   is  the   cause   of anemia?

Explanation

Blast cells are immature WBC. Presence of 32% of Blast cells in the analysis indicates an acute form of leukemia. Less than 10% of blast cells will indicate a chronic form. This patient also has an anemia (post hemorrhagic form) indicated by the low level of RBC. N/B, when there is leukemia, most of the immature cells will be recruited to produce the cancer cells which will cause a reduction in most of the other cell types especially RBC’s.

Megaloblastic anemia is seen in vitamin B12 deficiency while chronic hemolytic anemia will show reticulocytes ( immature RBCs) on blood film unlike leukemia that show immature wbc.

3. A  regional  cardiologist  is tasked  with the development of a plan for medioprophylactic measures aimed at decrease of cardiovascular mortality.  What  measures should  be planned for secondary prevention?

Explanation

Primary prevention of a disease is aimed at preventing the onset (start of a disease) ie, the disease is yet to occur so we aim at preventing it from happening via optimization of lifestyle and living conditions etc.

Secondary prevention is aimed at preventing the progress of a disease ( early diagnosis and treatment) ie, the patient already has the disease, so we are more concerned on stopping it from progressing and preventing its complications.

 
4. A  woman  complains  of muscle  weakness and   general   fatigue,   dyspnea,   vertigo,   brittleness   of  her  hair  and  nails,  an  urge  to eat  chalk.  Anamnesis states  uterine fibroid. Common   blood  analysis:  erythrocytes -  2,8

Т/l, Hb- 80 g/l, color index - 0,78, anisocytosis, poikilocythemia,  serum   iron  -  10  mcmol/l.What diagnosis is most likely?

Explanation

Anemia is a condition characterised by too few RBCs . In iron deficiency anemia, we observe the presence of low hemoglobin levels coupled with a low color index; recall that iron is responsible for the characteristic red color in Rbc and a deficiency in Iron content in blood will lead to a decreased color index. In this patient, together with the RBC level, the color index and hemoglobin levels are also low ( norm Color index 0.85- 1.05,  Hemoglobin, blood Male: 135-175 g/L Female: 120-160 g/L); Anisocytosis (  red blood cells of varying sizes on your blood smear.) and poikilocytosis are also key findings in iron deficiency anemia ( red blood cells of varying shapes on your blood smear).

B12 deficiency will lead to the presence of megaloblasts in blood smear with the patient experiencing some neurological disturbances.

In aplastic anemia, there will be a deficiency in all types of blood cells. This is because, in this condition, the body stops producing sufficient blood cells.

5.

A 24-year-old patient visited a doctor complaining  of enlargement of his submaxillary   lymph   nodes.   Objectively:    submaxillary,   axillary   and   inguinal   lymph   nodes are enlarged. Chest X-ray shows: enlarged lymph nodes of mediastinum. Blood test: erythrocytes - 3, 4 · 1012 /l, Hb- 100 g/l, blood colour  index  -  0,88,  platelets  -  190 · 109 /l, leucocytes - 7, 5·109 /l, eosinophiles - 8%, band neutrophiles - 2%,  segmented neutrophiles - 67%, lymphocytes  - 23%, ESR-  22 mm/hour. What  test  must  be  prescribed to  verify  the

cause of lymphadenopathy?

Explanation

 

A lymph node biopsy removes lymph node tissue to be looked at under a microscope for signs of infection or a disease, such as cancer. Lymph nodes in healthy people are usually hard to feel. But lymph nodes in the neck, armpit, or groin can get bigger and become tender. Swollen lymph nodes usually mean an infection. Open ( excisional) and needle biopsies are major methods of obtaining lymph nodes for diagnosis. open biopsy is a surgical procedure in which the entire lymph node is removed and taken for analysis; in needle biopsy a needle aspirate of the affected lymph node is taken.

6. An 8-year-old boy developed a temperature of 37, 5o C  two  days  after  his recovery  from the case of URTI. He complains  of suffocation, heart  pain.  Objectively:  the  skin is pale, tachycardia, the  I heart  sound  is weakened, short  systolyc murmur in the  4th intercostal area  near  the  left  edge  of  the  breastbone. What  heart  disorder such  clincal presentation is characteristic of?

Explanation

Nonrheumatic carditis refers to inflammation and may be caused by many infections, toxic (cocaine, ethanol, heavy metals) or idiopathic processes (giant cell myocarditis, diabetes mellitus, sarcoidosis, systemic lupus erythematosus, thyrotoxicosis), affecting the myocardium with or without associated systemic manifestations of disease process or involvement of the endocardium or pericardium;

The most common manifestation is heart failure, although arrhythmias and sudden death may be the 1st detectable signs. Its manifestations are age dependent:

in early infancy- viral myocarditis often occurs as a acute, fulminant disease;

in toddlers, young children - it occurs as an acute myopericarditis;

in older children, adolescents – it is often asymptomatic.

Primary rheumatic carditis; one of the succeeding signs of Rheumatic fever ( caused by a group A beta hemolytic streptococci infection).

7. A 32 year old patient complains  of cardiac irregularities, dizziness,  dyspnea  at  physical exertion. He has never suffered from such condition before.  Objectively:  Ps- 74/min., rhythmic.   BP-  130/80  mm  Hg.  Auscultation  revealed  systolic  murmur  above   aorta, the   first   heart   sound   was   normal.   ECG showed  hypertrophy of the  left ventricle,  signs of repolarization disturbance in the I , V5 and  V6  leads.  Echocardiogram revealed that interventricular septum  was 2 cm. What is the most probable diagnosis?

Explanation

The key findings here are the results from the ECG and the echocardiogram. Notice that the left ventricle has undergone a hypertrophy and the interventricular septum is increased in size ( norm 0.6-1.1cm). Hypertrophic cardiomyopathy is a condition in which the heart becomes enlarged without any underlying disease/pathology; this reduces the heart’s pumping ability leading to  cardiac irregularities, dizziness, dyspnea etc.

 

Aortic stenosis is the narrowing of the walls of the aorta thereby restricting blood flow through the aorta. Chest pain, lightheadedness, difficulty walking are key symptoms

8. A 35-year-old patient’s wound with suppurative  focus   was   surgically   cleaned. On  the  8th day after  the  surgery  the  wound cleared  from  its purulo-necrotic content and granulations appeared. However, against  the bacground of antibacterial therapy the  body temperature keeps  at 38,5-39,5oC . There  are chills, excessive sweating, euphoria, heart rate is 120/min. What  complication of local pyoinflammatory process can it be?

Explanation

Sepsis is simply an inflammatory immune response triggered by infection. The above patient most likely got an infection during the surgical process; Fever, chills, excessive sweating, tachycardia are all signs of the body’s reaction to an infection. N/B sepsis can be of bacterial, fungal, protozoal or viral cause and the patient’s non-response to antibacterial therapy may indicate a non bacterial origin.
9. A  60-year-old   woman   has  been   suffering  from  arterial hypertension for  15 years. After  recurrent stroke  she started complaining of unmotivated bad mood,  problems with attention concentration; she  forgets  to  close the  entrance  door,  cannot   recall  events   of the  past  day.  Computer  tomography shows areas of postinfarction changes in the cortical postfrontal areas.  What  is the most probable diagnosis?

Explanation

Observe that the patient has a history of stroke which from the CT scan result created postinfarction changes in the cerebral cortex. This patient also presents with problems of reasoning,memory, decision and thought which are signs seen in patients with dementia (a chronic organic mental disorder characterised by impairment in intellectual function,memory and personality). We can conclude that the dementia is as a result of disturbed brain blood supply (vascular dementia).

Alzheimer's disease is a type of dementia that causes problems with memory, thinking and behavior. It is a progressive disorder that causes brain cells to waste away (degenerate) and die. Pick’s disease is also a type of dementia but it is specific because it affects mainly the Frontotemporal lobe.
10. Clinic of a research instutute for occupational diseases  examined  a worker  who works at  a concentration plant  and  diagnosed him with chronic dust bronchitis. The case is investigated by  a  commission  including  the representatives  of:  the   plant,   clinic,  local SES,  department of  Social  Insurance Fund, trade  union.  According to the ”regulation on investigation of. . . ”, the commission should be headed by the representative of the following authority:

Explanation

This committee should be headed by the local SES ( state emergency committee) who are responsible for providing assistance especially in emergency situations to their immediate environment.
11. A 37-year-old woman complains of headaches,  nausea,   vomiting,   spasms.   The onset  of the disease  occurred  the  day before due  to  her  overexposure  to  cold.  Objectively:  fever  up  to  40o C ;  somnolence;  rigid neck;  Kernig’s  symptom   is  positive  on  the both sides; general  hyperesthesia. Blood test: leucocytosis,   increased   ESR.   Cerebrospinal fluid is turbid,  yellow-tinted. What changes of the cerebrospinal fluid are most likely?

Explanation

 
  1. A positive kernig sign indicates a form of meningitis or subarachnoid hemorrhage; for more information CSF findings are taken; the analysis shows no sign of hemorrhage,  but appears turbid and yellow tinted which are signs of a bacterial meningitis. Examination of the CSF in patients with acute bacterial meningitis reveals the characteristic neutrophilic pleocytosis ( a neutrophil count in the CNS greater than 50%). Lymphocytic pleocytosis is found in viral encephalitis.

12. A  48-year-old  woman  complains  of pain in   the   thoracic   spine,   sensitivity   disorder in  the  lower  body,  disrupted motor   function  of  the   lower   limbs,  body   temperature rise  up  to  37, 5o C .  She  has  been  suffering from  this  condition for  3  years.  Treatment by various specialists was ineffective. X-ray reveals  destruction  of  adjacent  surfaces  of the VIII and IX vertebral bodies. In the right paravertebral area at the level of lesion there is an additional soft tissue  shadow.  What  diagnosis is most likely?

Explanation

Also known as Pott’s disease ( a major form of extrapulmonary tuberculosis) , lumbar tuberculous spondylitis is a medical condition caused by the hematogenous spread of tuberculosis mostly from the lungs; these spread contents are latter deposited in the spine- mostly the lower thoracic and upper lumbar vertebrae; they can also affect the intervertebral disc manifesting mainly as a combination of osteomyelitis and arthritis that usually involves more than 1 vertebra. Multiple sclerosis is a demyelinating disorder. In this condition, the body's immune system attacks the myelin cover present in the nervous system thereby disturbing communication between brain and the rest of the body; leading to signs such as numbness, visual impairment, gait etc
13. A pregnant woman is 28 years old. Anamnesis: accelerated labor complicated by the  II  degree  cervical  rupture. The  following two  pregnancies resulted in spontaneous abortions at  the  terms  of 12 and  14 weeks. On  mirror   examination:  the  uterine cervix is scarred  from  previous  ruptures at 9 and  3 hours,  the  cervical  canal  is gaping.  On  vaginal examination: the  cervix is 2 cm long, the external  orifice is open 1 cm wide, the internal orifice is half-open;  the  uterus  is enlarged to the 12th week of pregnancy,  soft, mobile, painless,  the  appendages are  without   changes. What diagnosis would you make?

Explanation

Realise that this patient is experiencing cervical changes that are more advanced in respect ( corresponding) to the duration of pregnancy. The word ‘gaping’ indicates that the walls of the cervix is weak and from vaginal examination, we observe the dilation and effacement of the cervix. Note that these changes are not due ( In a normal pregnancy, dilation and effacement occurs in response to uterine contractions). These changes in the cervix are typically seen in a condition termed ‘cervical weakness’ or ‘cervical insufficiency’ ie., cervical dilation and effacement occurring in preterm pregnancy. 

A cervical pregnancy usually terminates during the first trimester and occurs when implantation takes place in the cervix.

Incipient abortion is a form of spontaenous abortion. It refers to the non induced fetal death before 20 weeks of gestation.

 
14.

A 56-year-old patient complains of pain in the  epigastrium after  eating,  eructation, loss of  appetite, slight  loss  of  weight,  fatigability. The  patient smokes;  no excessive alcohol consumption. Objectively:  pale  mucosa,  BP-110/70 mm  Hg.  The  tongue   is  ”lacquered”. The  abdomen is  soft,  sensitive  in  the  epigastric area. Blood test: erythrocytes - 3,0 T/l, Hb-  110 g/l, color  index  - 1,1; macrocytosis; leukocytes  - 5,5 g/l, ESR-  13 mm/hour.   On fibrogastroduodenoscopy: atrophy   of  fundic mucosa. What pathogenesis does this disorder have?

Explanation

 

The key finding her is the atrophic changes in the FUNDUS of the stomach from the fibrogastroduedenoscopy. Atrophic gastritis in a form of chronic gastritis in which the gastric glandular cells are replaced with intestinal and fibrous tissues. Note that the type A form of atrophic gastritis is caused by the autoimmune destruction of these cells and occurs in the FUNDUS; while the type B form is caused by Helicobacter pylori persistence- this form affects the ANTRUM of the stomach. This patient presents with atrophy of the fundic mucosa thereby indicating antibodies against parietal cells are the causative agent.

15.

A 26-year-old  secundipara at 40 weeks of gestation   arrived  at  a  maternity ward  after the  beginning  of  labor  activity.  The  bursting  of  waters   occurred  2  hours   prior.   The fetus  was in a longitudinal lie with  cephalic presentation. Abdominal circumference was 100  cm,  fundal  height  -  42  cm.  Contractions  occurred   every  4-5 minutes   and  lasted 25 seconds each. Internal obstetric examination revealed cervical effacement, opening by 4cm. Fetal bladder was absent.  Fetal head was pressed against the pelvic inlet. What complication arose in the childbirth?

Explanation

Recall that labor physiologically is divided into 3 stages;

In the first stage the cervix opens to full dilation to allow the head to pass through  The second stage is from full dilation to delivery of the fetus. The third stage lasts from delivery of the fetus to delivery of the placenta. Labor often lasts between 12 and 14 hours – or longer – for first-time mothers, but is usually shorter in subsequent births.

The 1st stage—is the longest stage of labor, and is divided into three separate phases:

The early ( latent) phase-  averaging 8 1 ⁄2 h in nulliparas and 5 h in multiparas; duration is considered abnormal if it lasts > 20 h in nulliparas or > 12 h in multiparas.

Active labor (the active phase)-  here, the cervix dilates to 7cm.  On average, the active phase lasts 5 to 7 h in nulliparas and 2 to 4 h in multiparas. The cervix should dilate 1.2 cm/h in nulliparas and 1.5 cm/h in multiparas.

Transition. The cervix dilates from 7 centimeters to 10 centimeters. This is usually the shortest stage of labor, but is often the most unpleasant.

If a woman's “water ―breaking” occurs in the first stage before/ until the active phase, it is called early amniorrhea.

In Primary uterine inertia, uterine contractions fail to be initiated while in the secondary form, uterine inertia ceases in between labor ( before completion)

16. Bacterial analysis of air in a living space in winter period by means of Krotov’s apparatus revealed that total number of microorganisms in 1 m3  of air was 7200. What  is the allowed number of microorganisms for the  air  to  be characterized as ”pure”?

Explanation

  1. Air cleanness estimation according to indexes of bacteriological research in different year periods.

 

Air estimation

Microorganisms Contents in 1m3 of air.

 

Generally

Streptococcus hemolyticus

Generally

Streptococcus hemolyticus

 

Summer period

Winter period

Clean

less 1500

less 16

less 4500

less 36

Moderately muddy

1500 - 2500

16-35

4500-7000

36-125

Muddy

more 2500

more 35

more 7000

more 125

17. A    24-year-old     patient    received     a puncture injury below the Poupart’s ligament accompanied by intense arterial bleeding. The best method  to temporarily stop the bleeding in the patient would be:

Explanation

Poupart ligament refers to the inguinal or groin ligament. Compression bands are also known as pressure bandages;  are used to control bleeding and encourage blood clotting without constricting normal blood circulation. Esmarch tourniquet is used to stop bleeding located on the limbs.
18. A 42-year-old  womahas beehospitalized  with  complaints  of intense  pain  attacks in  the  lumbar   and  right  iliac  areas,  which irradiate to the vulvar lips, frequent urination, nausea.  The  pain  onset  waacute.  Objectively:  the  abdomen isoft,  moderately painful in the right subcostal area, costovertebral angltenderness on the  rightCommon  urinanalysis: specific gravity  - 1016, traces  of protein, leukocytes  - 6-8 in the  vision  field, erythrocytes - 12-16 in the vision field, fresh. What diagnosis can be made?

Explanation

19. A    29-year-old    patien works    a a motor  mechanic.  Anamnesis shows frequent exposure to cold, exacerbation of chronic bronchitis  attended  by  coug with   relativly small  amount   omucopurulent sputum, subfebrility,  sometimes joineby hemoptysis and pain in the right side of chest. Breathing is vesicular.  X-ray  shows shadows  ansharp decrease in size of the lower lobe distinctly visible on the X-ray image as a streak 2-3 cm wide situated at the angle from lung root to the frontal  costodiaphragmatic recessThe  most likely diagnosis is:

Explanation

20. A 62-year-old  patient has beehospitalized  with  complaint of  pain  in  the  thorax othe  right  during  breathing, dyspnea,  dry cough. Ten days ago he slipped  and fell hitting his right  side.  On  examintaion: the  patient lies on the left side. The right side of the thora lags  during  breathing.  On  the  right there are crepitation and pain in the III-IV ribs. Dullnesof percussion  sound and sharply diminished   breath sounds  can  be  observed. On X-ray: signs of exudate, fracture of the III- IV ribs. On pleurocentesis: blood is detected. Choose the further tactics:

Explanation

21.

A 29-year-old female patient complains of dyspnea,  heaviness  and  chest pain  on the  right, body temperature rise up to 37, 2o C . The disease is associated with a chest trauma received 4 days ago. Objectively:  skin is pale and moist.  Ps- 90/min., regular.  Palpation reveals a dull sound on the right, auscultation reveals significantly   weakened  vesicular   breathing. In  blood:  RBCs-  2, 8 · 1012 /l, colour  index  - 0,9, Hb-  100 g/l, WBCs-  8, 0 · 109 /l, ESR-  17 mm/hour.  What results of diagnostic puncture of the pleural cavity can be expected?

Explanation

From anamnesis, the patient is said to have experienced a chest trauma which is the reason behind the presented symptoms. Observe that the RBC level is low ( norm Female - 3.5-5.5 x 10^12/l), this reduced amount is most likely as a result of bleeding from the chest trauma ( post hemorrhagic anemia). This implies that when the pleural puncture is taken, signs of hemorrhage are going to be found on the punctate ( a hemorrhagic punctate).

 

A transudate is due to high capillary pressure and has protein levels below 25g/l or 2.5g/dl while an exudate is mostly present in inflammatory processes and its protein content is above 25g/l or 2.5g/dl.

22. Caries  morbidity rate  is 89% among residents  of a community. It  is determined that fluorine  content in  water  is 0,1 mg/l.  What preventive measures should be taken?

Explanation

 

Dental caries is a bacterial disease that begins with demineralization of the outermost dental enamel and progresses, if not halted, can lead to loss of tooth substance and infection of the dental pulp. Fluoride acts in several ways to prevent caries. The principal action is thought to be that fluoride in dental plaque inhibits the initial demineralization of enamel, and then promotes remineralization of early lesions. Fluorine is gotten from drinking water and food materials. Adequate amount of fluorine in water should be from 0.7-1.5mg/l

23.

Examination of a group  of persons  living on the  same  territory revealed the  following common  symptoms:  dark-yellow  pigmentation of the  tooth  enamel,  diffuse  osteoporosis of bone  apparatus, ossification  of  ligaments and joints, functional disorders of the central nervous system. This condition may be caused by the excessive concentration of the following microelement in food or drinking water:

Explanation

Presence of dark-yellow pigmentation of the tooth enamel, diffuse osteoporosis of bone apparatus, ossification of ligaments and joints are specific symptoms that indicate Fluorosis. This is a condition that occurs as a result of excessive intake of fluorine.
24. In a pre-school educational establishment the menu consists of the following dishes: milk porridge from buckwheat, pastwith minced meat, cucumber salad, kissel (thin berry jelly), rye bread. What dish should be excluded from the menu?

Explanation

25.

An infant has been born at the 41st week of gestation.  The pregnancy  was complicated with  severe  gestosis  of the  second  semester. The weight of the baby is 2400 g, the height is 50 cm. Objectively: the skin is flabby, the layer of  subcutaneous fat  is  thin,  hypomyotonia, neonatal  reflexes   are   weak.   The   internal organs  are  without  pathologic changes.  This newborn can be estimated as a:

Explanation

Norms;  gestation period- 280 days or 40 weeks up to 42 weeks

              birth weight- 3-3.5kg ( 3000-3500g)

             Body length- 45-55cm

The baby in question underwent a full term gestation period (41 weeks) but is underweight ( 2400g). This underweight is due to complications that occurred during gestation as stated “The pregnancy was complicated with severe gestosis in the second semester”. We can therefore conclude that the baby who had a full term gestation had some intrauterine growth restrictions.

A premature infant is one which is born before 37 weeks of gestation.

 

A post mature infant is one born after 42 weeks of gestation.

26. A    patient   suffering    from    infiltrative pulmonary tuberculosis was prescribed streptomycin, rifampicin,  isoniazid, pyrazinamide,  vitamin C. One month  after the beginning  of  the  treatment the  patient started complaining  of reduced hearing  and tinnitus. What drug has such a side effect?

Explanation

Tuberculosis is a disease caused by an acid fast bacteria, Mycobacterium tuberculosis. Streptomycin, Rifampicin, Isoniazid, pyrazinamide and ethambutol are very active drugs chosen for its treatment. The following are lists of side effects associated with these drugs:

Streptomycin- vestibular and auditory ( ototoxicity) dysfunction, non oliguric renal failure

Rifampicin- red/ orange discoloration of body fluids e.g. tears, urine; flu-like symptoms, hepatitis, diarrhea, thrombocytopenia.

Isoniazid- hepatitis, peripheral neuropathies

Pyrazinamide- asymptomatic hyperuricemia, joint pain, rash

Ethambutol- retrobulbar optic neuritis

27.

A woman  has developed sudden  thoracic pain  on  the  right  with  expectoration of  pink sputum  and  body  temperature rise up to 37, 7o C  on  the  4th  day after  the  surgery  for cystoma of the right ovary. On lung examination: dullness  of the  lung sound  on the  lower right is observed. Isolated moist crackles can be auscultated in the same area. What complication is the most likely?

Explanation

Observe that this patient had a surgery for cystoma of the right ovary 4 days ago; Pulmonary embolism is one major complication that can occur in individuals after surgery-This refers to the obstruction of an artery in the lungs by a travelling substance eg, air, fat etc. the blocking of an artery in the lungs will lead to the creation of an  infarct in the affected area ( an infarct is an are of dead tissue due to loss of blood supply.
28. A 58-year-old  patient was delivered to an admission room with complaints  of pain in the thorax  on  the  left.  On  clinical  examination: aside  from  tachycardia (102/min.)  no  other changes.  On  ECG:  pathologic wave  Q  in I, аVL, QS in V1, V2, V3 leads and ’domed’ ST elevation with negative  T. What  diagnosis  is most likely?

Explanation

From the ECG, the presence of pathological Q wave and an ST elevation are key findings in individuals with prior myocardial infarction. To differentiate the part of the heart affected, we use the leads in which changes occur. QS deflection occurs in leads V1-V3 which are leads representing the anterior surface of the heart.

Anatomical relations of leads in a standard 12 lead electrocardiogram

  • II, III, and aVF: inferior surface of the heart

  • V1 to V4: anterior surface

  • I, aVL, V5, and V6: lateral surface

  • V1 and aVR: right atrium and cavity of left ventricle

  • Posterior MI will present with ST segment depression (not elevation) in the septal and anterior precordial leads (V1-V4) and ST elevation in the posterior leads of a posterior ECG (leads V7-V9). 

29. A 48-year-old  woman  has thermal burns of  both  hands.  The  epidermis  of  the  palms and backs of her hands is exfoliating,  and blisters filled with serous liquid are forming. The forearms are  intact.  What  diagnosis  is most likely?

Explanation

  1.  rule of “nines” – area of different areas of the body is proportional: anterior surface of the trunk – 18 %, posterior – 18 %, lower limb – 18 %, external genitals – 1 %);

   rule of  “palm”. It is used if burns are limited and located on different areas of the body. According to the rule of palm takes 1 % of the skin surface.

Division of the burns on superficial (I, II, IIIA st.) and deep (IIIB-IV st.)

I stage – hyperemia of the skin

II stage – separation of epidermis with formation of bullas

III A stage – necrosis of superficial layers of the skin with saving of bulbs hair, sweat glands and sebaceous glands.

IIIB stage – necrosis of all the derma

IV stage – necrosis of the skin and underlying tissues.
30. A 39-year-old patient complains of morning headache, appetite loss, nausea,  morning vomiting,  periodic  nasal  haemorrhages. The patient had a case of acute  glomerulonephritis at the age of 15. Examination revealed rise of arterial pressure up to 220/130 mm Hg, skin haemorrhages on his arms and legs, pallor  of skin and mucous membranes. What biochemical parameter is most  important for making diagnosis in this case?

Explanation

 

Creatinine clearance test is one of the tests done to check the Renal function. It involves the measurement of the amount of plasma cleared of creatinine per unit time.The above patient presents with an acute glomerulonephritis and for biochemical analysis, the blood creatinine level is most informative. Another important test is the checking of the glomerular filtration rate.

31. A     30-year-old     patient,    who     has been suffering from headaches, suddenly developed extreme headache after  lifting  a heavy  load,  as  if he  had  been  hit  over  the head.   Nausea,   vomiting,   and   slight   dizziness are observed. In a day he developed pronounced meningeal   syndrome and  body temperature up to 37, 6o C . A doctor  suspects subarachnoid hemorrhage.  What   additional examination is necessary  to  confirm  this  diagnosis?

Explanation

It is well noted that on examination, the patient presents with ‘Meningeal Syndrome’ (stiff neck, sensitivity to light etc). To understand the cause of these meningeal signs, a lumbar puncture is carried out inorder to obtain the cerebrospinal fluid for analysis.
32.

A worker  of a blowing shop complains  of headache, irritability,   sight  impairment - he sees everything  as if through  a ”net”.  Objectively: hyperemic sclera, thickened cornea, decreased opacity  of  pupils,  visual  acuity  is 0,8 in the  left  eye, 0,7 in the  right  eye. The worker uses no means of personal protection. What diagnosis is most likely?

Explanation

Cataract is the opacification ( clouding ) of the lens which leads to a decrease in vision ( visual acuity). The visual acuity in adults is 60/60 or 6/6 which equals 1. Note that the patient in view has reduced visual acuity in both eyes and has a net like vision ( cloudy); these signs indicate presence of cataract.

In conjunctivitis, we see red eyes ( inflammation of the transparent membrane- conjunctiva), it doesn't affect visual acuity.

 

Keratitis is an inflammation of the Cornea. Symptoms include; eye redness, photophobia, eye pain coupled with a reduced visual acuity.

33. A 45-year-old woman is undergoing treatment for  active  rheumatism, combined mitral valve failure. During her morning procedures she  suddenly  sensed  pain  in the left hand,  which was followed  by numbness. Pain  and  numbness continued to  aggravate. Objectively:  the  skin of the  left hand  is pale and comparatively cold. Pulse in the hand arteries  is  absent   along  the   whole  length. What treatment tactics is most efficient?

Explanation

 

From the description of the current state of the patient’s left hand ( pale, numb, pulseless, cold), we can say there is a case of restricted blood flow to the particular region. From anamnesis, this patient has a case of rheumatism which increases the chances of formation of a thrombi or emboli ( deep venous thrombosis, pulmonary embolism etc). The obstruction of blood flow is most likely due to a substance that has traveled and lodged in this area ( an emboli). Because of the severity of the current state of the limb, an urgent embolectomy ( a surgical removal of an embolus) should be carried out. In a less severe case in which pulse can still be felt and the limb is slightly active, fibrinolytics and anticoagulants will be the procedure of choice.

34. A 10-year-old boy had a case of viral hepatitis type B four years ago. Currently the assumption was made about  the formation of hepatic  cirrhosis  in the  patient. What  additional investigation can clarify the diagnosis?

Explanation

Individuals with liver cirrhosis, alcoholic hepatitis and other chronic liver diseases are most likely to develop hepatorenal syndrome. Hepatorenal syndrome is a medical condition in which the kidneys are progressively damaged  usually due to an increase in pressure in the portal vein, build up of toxins; these changes take place as a result liver insufficiency. A renal needle biopsy (percutaneous biopsy) is the most common method of obtaining a kidney specimen. In this procedure, a thin needle is used in obtaining kidney materials for microscopic examination. Note that the question asks about an ‘Additional investigation’- patients with liver cirrhosis are usually examined to rule out the development of hepatorenal syndrome.
35. A  40-year-old   patient  has  acute   onset of  disease  caused  by  overexposure to  cold. Temperature  has   increased    up   to   39o C . Foul-smelling   sputum   is  expectorated during coughig. Various moist crackles can be auscultated above  the 3rd segment  on the right. Blood test: leukocytes  - 15, 0 · 109 /l, stab neutrophils - 12%, ESR-  52 mm/hour.  On X- ray: in the 3rd segment  on the right there  is a focus of shadow 3 cm in diameter, low density, with fuzzy smooth  margins  and  a clearing  in its center.  What  disease  is most  likely in the given case?

Explanation

Pneumonia is an infection of the lung tissue that leads to the inflammation of the alveoli, making breathing difficult. From examination: Crackles are heard on auscultation; crackles on auscultation are often associated with inflammation or infection of the small bronchi, bronchioles, and alveoli. On blood film, there is an increase in WBC, ESR and stab neutrophils which indicate an acute bacterial infection. Recall that a lung abscess is a complication of Pneumonia ( pus filled cavities greater than 2cm ‘liquefactive necrosis of lung tissue’); these abscesses can be seen in the x ray results and are the reason behind the presence of foul smelling sputum on physical examination. Recall that a cyst is a fluid filled cavities- Cystic echinococcosis ( hydatid cyst) is caused by infection by the E. granulosus. the cysts are characteristically filled with clear fluid, spherical and located in one area of the body.

Echinococcosis Hydatid Cyst

36. An  electro-gas   welding  operator working at a machine  workshop performs welding and  cutting  of metal,  which  is accompanied by intense  UV-radiation. His welding station is equipped with  efficient  mechanical ventilation.   What   occupational  disease   is  most likely  to  develop  in  an  electro-gas   welding operator?

Explanation

 

The main risk amongst welders is the inflammation of the cornea and conjunctiva, commonly known as 'arc eye' or 'flash'. This is caused by the irritation of the eyes by the ultraviolet rays produced. Other possible hazards include skin inflammation, cancer.

37.

A 48-year-old  patient was found  to have diffuse enlargement of the thyroid  gland, exophthalmia, weight loss of 4 kg in 2 months, sweating.   Objectively:   HR-   105/min.,   BP - 140/70  mm  Hg.  Defecation  act  is  normal. What kind of therapy is recommended in this case?

Explanation

The patient experiences an enlarged thyroid gland, tachycardia, loss of weight  and exophthalmos which are symptoms of Hyperthyroidism ( thyrotoxicosis). For treatment Mercazolil (thiamazole) is administered. Propranolol is a beta blocker and used in heart related issues while thyroxine is indicated for hypothyroidism.
38. A woman  addressed a doctor  with complaints  of increased  body temperature up to 37, 8o C and moderately sore throat for the last  3 days.  Objectively:   mandibular  lymph nodes are enlarged up to 3 cm. Palatine tonsils are hypertrophied, covered  with grey coating that  spreads  to the uvula and anterior pillars of the fauces. What diagnosis is most likely?

Explanation

 

The key finding here are grey white coatings on the tonsils that cannot be removed ( or bleeds when removed); these are the main findings of a diphtheria infection ( causative agent -Corynebacterium Diphtheriae which produces an Exotoxin ie.,diphtheria toxin).

N/B The posterior pharynx of patients with Infectious Mononucleosis (EBV infection)  is often uniformly erythematous.

39. A 48-year-old  man complains  of constant pain  in  the  upper  abdomen, predominantly on the left, which aggravates after eating, diarrhea,  loss  of  weight.   The   patient  has alcohol  use  disorder. Two  years  ago  he  had a  case  of acute  pancreatitis. Blood  amylase is 4 g/hour·l. Feces analysis: steatorrhea, creatorrhea. Blood sugar is 6,0 mmol/l. What treatment should be prescribed?

Explanation

This patient has a case of acute Pancreatitis. Recall that the pancreas has both endocrine and exocrine functions; for the exocrine part, it produces enzymes that aid in digestion such as amylase, lipase, trypsin. Absence of these enzymes will lead to symptoms such as steatorrhea  (fats in feces), creatorrhea (undigested muscles in feces) etc. Panzinorm Forte is a combination of digestive enzymes. These enzymes are normally produced by the pancreas and are important for digesting fats, proteins, and sugars.

Panzinorm Forte is used to replace digestive enzymes when the body does not have enough of its own. Certain medical conditions can cause this lack of enzymes, such as cystic fibrosis, pancreatitis, pancreatic cancer, or pancreas surgery.

Panzinorm Forte may also be used to treat a condition called steatorrhea (loose, fatty stools).

 

Pirenzepine is an M1 selective antagonist used in treatment of peptic ulcer ( it reduces gastric acid secretion and muscle spasm). Contrykal is a protease inhibitor and is used in reducing blood loss in surgical procedures. Drotaverine is an antispasmodic drug used for cervical dilation in childbirth

40. In  10  hours  after  eating  canned mushrooms  a  27-year-old   patient  has developed   diplopia,    bilateral   ptosis,    disrupted swallowing,  shallow  breathing with respiratory rate 40/min., muscle weakness, enteroparesis. What measure should be taken first?

Explanation

 

This patient is experiencing a mushroom poisoning; amongst the listed symptoms, notice the presence of shallow breathing with respiratory rate 40/min ( norm 12-20/min)  to avoid a complete respiratory failure, tracheal intubation with mechanical ventilation should be performed on the patient. Intubation is done when one cannot maintain their airway on their own due to anesthesia or illness. Notice the question says what should be done first- a gastric lavage and detoxification therapy can follow next but maintenance of respiration should be the first priority.

41. A 32-year-old patient complains of reddening, burning, and sensation  of a foreign body in the right eye. The disease is acute. On examination: visual  acuity  of  the  both  eyes is 1,0. In  the  right  eye  there  are  hyperemy and  swelling  of  the  conjunctiva, superficial injection.  There  is purulent discharge  in the conjunctival sac. The cornea is clear. The color and pattern of the iris are uncanged,  the pupil is mobile. What diagnosis is most likely?

Explanation

The key finding here is that visual acuity in both eyes is 1,0 which implies that vision is not affected. From the listed options, conjunctivitis ( red/pink eye) does not affect visual acuity.  

Conjunctivitis refers to the inflammation of the transparent membrane that lines the eyelid and the sclera; this inflammation is due to an infection ( bacterial, viral, chlamydial etc). Symptoms include; burning and itchy sensation, sticky mucus discharge, conjunctival injection ( redness of the eye) etc.

 

Iridocyclitis is the inflammation of the uveal tract ( the iris, ciliary body and choroid), glaucoma involves the increase in intraocular pressure that gradually leads to the loss of vision. Dacryocystitis involves an infected lacrimal sac and an obstruction in the lacrimal duct.

42. A patient is on the sick leave for 4 months continuously from the date of injury. The treatment is going to last for another 1-2 months.   Who  has  the  right  to  extend   the duration of medical  certificate  for  this  patient?

Explanation

The right of prolongating a medical certificate (in this case a sick list) of invalidity alongside with the medical treatment by the doctor is granted by:

• Chief medical officer (assistant of the chief medical officer on expertise of ability for work);

• Manager of department;

• Medical-consulting commission also known as medical expertise committee

 

The medical-consulting commission (MCC) is formed in case the doctors staff is above 15 doctors.

43.

Monthly dysentery morbidity in the region given in absolute figures is as follows: January - 6; February - 9; March - 11; April - 10; May - 16; June - 23; July - 19; August - 33; September - 58; October - 19; November - 11; December - 5. Annual total  is 220 cases. What  graphic presentation would provide the best visual for

monthly   deviations  of  dysentery  morbidity from the average?

Explanation

Radar charts are also known as spider, polar, web charts or star plots); they are used as a way to visualize multivariate data. They are used to plot one or more groups of values over multiple common variables. They do this by giving an axis for each variable, and these axes are arranged radially around a central point and spaced equally.

A cartogram uses a map to differentiate how a variable is distributed

 
44. A  30-year-old  woman  complains  of pain in  the  heart   area  (”aching,   piercing  pain”) that  arises primarily  in the morning  hours  in autumn and  spring.  Pain  irradiates into  the neck, back, abdomen and is attended by rapid heart  rate  and low vital tonus. This condition occurs  independently  from  physical  exertion.  In  the  evening  her  condition improves. Somatic  and neurologic  state  and ECG  have no  pathologies. What  pathology is likely  to result in such clinical presentation?

Explanation

Somatoform disorders - multiple, recurrent and frequent somatic complaints requiring medical attention without association with any physical disorder. The above patient’s heart ache is not related to any physical activity and occurs at a particular period of the year, 

 

Hypochondriacal Disorder is characterized by a persistent preoccupation and a fear of developing or having one or more serious and progressive physical disorders. Physician physical examination does not reveal any disorder, but the fear and convictions persist despite the reassurance.

45. A   full  term   baby   born   from   the   1st noncomplicated pregnancy with complicated labor  was diagnosed with cephalohematoma. On  the  2nd  day  of life the  child  developed jaundice; on the 3rd day of life there appeared neurological changes: nystagmus,  Graefe syndrome. Urine  is yellow, feces are  golden- yellow.  The  mother’s  blood  group  is А  (II) Rh, the  child’s - А  (II)  Rh+.  On  the  3rd day the results of the child’s blood test are as follows: Hb- 200 g/l, erythrocytes - 6, 1 · 1012 /l, blood   bilirubin   -  58  mcmol/l   due   to   the presence of its unconjugated fraction, Ht- 0,57. In this case the jaundice is caused by:

Explanation

Although the Rh groups of the mother and child are different, the presence of normal erythrocyte and hemoglobin levels from the blood test rules out the option of hemolytic disease of the newborn. A cephalohematoma is a traumatic subperiosteal hematoma that occurs in the periosteum of an infant’s skull bone underneath the skin. Occurs mostly because of increased friction between infants cranium and maternal pelvis or forceps ( during assisted delivery). Infants with cephalohematoma are also at a heightened risk for developing jaundice, because as the blood breaks down, the levels of bilirubin increase. In these instances, and if the bilirubin is excessively high, treatment may include phototherapy.
46.

On  the  4th  day  after  recovering  from  a cold a patient was hospitalized with complaints  of  solitary  spittings  of  mucoid  sputum. On  the  2nd day there  was a single discharge of about  250 ml of purulent blood-streaked sputum.   Objectively:   the   patient’s  condition  is moderately severe.  Respiratory rate  - 28-30/min., Ps- 96/min., BP-  110/70 mm  Hg. Respiration above  the  left  lung  is vesicular, weak  above  the  right  lung.  There  are  various moist crackles  above  the lower lobe and amphoric  breath near  the  angle  of  scapula. What is the most likely diagnosis?

Explanation

Lung abscesses are usually a complication of pneumonia.Lung abscess is defined as necrosis of the pulmonary tissue and formation of cavities containing necrotic debris or fluid caused by microbial infection.  Lung abscesses can be classified based on the duration and the likely etiology. Acute abscesses are less than 4-6 weeks old, whereas chronic abscesses are of longer duration
47. A   46-year-old   patient  with   temporarily undetermined diagnosis was prescribed pleurocentesis based  on the results  of the X- ray. The puncture yielded 1000 ml of a liquid with  the  following  properties: clear,  specific gravity - 1,010, protein content - 1%, Rivalta’s test is negative,  erythrocytes - 2-3 in the field of vision. What  disorder are these  pathologic changes characteristic of?

Explanation

Rivalta’s test is a procedure used in differentiating the type of effusion i.e., transudate or exudate. It involves the mixture of a drop of the effusion with acetic acid and distilled water;If the drop dissipates ( disappears), the test is negative, indicating a transudate. If the drop precipitates, the test is positive, indicating an exudate. A transudate is a filtrate, it is typically a clear fluid with a low protein and cell content. Causes include increased capillary hydrostatic pressure (e.g., pleural effusion in congestive heart failure) and/or decreased capillary oncotic pressure (e.g., edema due to lack of albumin).

 

An exudate is a  A protein-rich fluid that occurs as a result of increased vascular permeability from inflammation. Analysis of the fluid reveals a cellular (cloudy) fluid with high protein and low glucose concentration. Can accumulate in cavities (e.g., pleural space, pericardial space).

48. A 14-year-old girl has been presenting with irritability  and tearfulness for about  a year. A year ago she was also found to have diffuse enlargement of the  thyroid  gland  (II  grade). This  condition was  regarded as  a  pubertal manifestation, the  girl did  not  undergo  any treatment. The girl’s irritability  gradually gave place to a complete apathy. The girl developed puffy face, soft tissues pastosity,  bradycardia, constipations. Skin  pallor  and  gland  density progressed, the skin developed a waxen hue. What disease can be suspected?

Explanation

Autoimmune thyroiditis (hashimoto’s thyroiditis) is a chronic disease in which the body creates antibodies which destroy the thyroid gland; it is characterised by symptoms of hypothyroidism ( bradycardia, increase in weight constipation etc). Notice that a year ago, she had a diffuse enlargement of the thyroid relatively due to a lymphocyte infiltration of the stroma. Diffuse toxic Goiter ( grave’s disease) also presents with an enlarged thyroid but is characterised by  signs of hyperthyroidism. Recall that a person with a cancer related issue in anamnesis, will present with an unexplained loss of weight within a short period.
49.

In  a  maternity hospital  a  newborn had been presenting with cough attacks  after eating. The child was discharged from the hospital on the 18th day due to a case of pneumonia. During  the  further 1,5 months  the  child had 2 cases of pneumonia. Periodically  there  are cough  attacks  after  eating,  especially  if the child lies on the  left side. Objectively:  the  II degree   hypotrophy, isolated   moist  crackles, dyspnea. Stool and diuresis are not disrupted. What diagnosis is most likely?

Explanation

Transesophageal fistula (TEF) is an abnormal connection between the esophagus and the trachea ie, the tube that leads to the stomach and that which leads to the lungs. It is characterised by continuous effusion of saliva from the mouth, choking and vomiting  with first feeding. Due to this abnormal connection, contents going to the stomach can easily get into the respiratory tract 

 

Mucoviscidosis ( cystic fibrosis of the pancreas) is a genetic disorder that occurs due to the mutation in CFTR gene. It is characterised by the production of thick and sticky mucus that can clog the lungs and obstruct the pancreas.

50.  26-year-ol woman wh gave   birth  7 months  ago,  habeen  suffering  from nausea mornin sickness,   somnolence  for the  last 2 weeks.  The  patient breasfeeds; no menstruation. She has been using no means of contraception. What  method  woulbe most efficient in clarification of the diagnosis?

Explanation

51. A  2-year-old  girl  has  a  medical  history of  recurrent  obstructive pneumonia. In  the lungs  various   moist   and   dry  crackles   can be  auscultated,  breath  sounds   are   diminished.  Sputum  is thick,  viscous  and  difficult to expectorate. Drumstick fingers and  physical developmental retardation are  observed. What preliminary diagnosis can be made?

Explanation

The key finding here is from the sputum ‘thick, viscous and difficult to expectorate’. This is characteristic of the disease cystic fibrosis ( pulmonary mucoviscidosis)  which refers to a form of genetic disorder that affects the CFTR gene. It basically affects  the cells that produce mucus, sweat and digestive juices ( the reason for the thick and viscous sputum); people with this pathology also encounter persistent lung infections. Note that drumstick fingers ( also known as clubbing of the nails)  are seen in individuals with cardiovascular diseases and lung pathologies. These lung diseases include;  lung cancer, lung infections, interstitial lung disease and cystic fibrosis.

 

In recurring bronchitis (chronic bronchitis), the individual experiences productive cough for about three months recurring in the space of two consecutive years. Bronchial Asthma is an allergic disease, a chronic obstructive pulmonary disease ( COPD) characterised by the swelling and narrowing of the airway and also the increased production of mucus.

52. After  a case of purulent otitis a 1-year-old boy has developed pains in the upper third of the  left thigh, body temperature up to 39o C . Objectively:  swelling of the thigh in its upper third  and smoothed out inguinal  fold. The limb is in semiflexed position. Active and passive movements are  impossible  due  to  severe pain. What diagnosis is most likely?

Explanation

Acute hematogenous osteomyelitis is an infection of the bone usually caused by a bacteria and common in children less than 5 years; typically affects the metaphysis of long tubular bones. Children present with fever, localized pain, swelling, and rarely erythema around a long bone, limited range of motion, and limping or refusal to bear weight or use an extremity (pseudoparalysis).

 

Brodie's abscess is a sub-acute form of osteomyelitis, presenting as a collection of pus in bone.  Classically, this may present after progression to a draining abscess extending from the tibia out through the skin.

53.

A  56-year-old  patient with  diffuse  toxic goiter has ciliary arrhythmia, heart rate is 110-120/min., arterial hypertension, BP is 165/90mm Hg. What  drug besides  Mercazolil  (Thiamazole) should be prescribed in this case?

Explanation

Propranolol is a drug indicated for the treatment of high blood pressure, a number of types of irregular heart rate, thyrotoxicosis, capillary hemangiomas, performance anxiety, and essential tremors. It is classified as a beta blocker (non- selective);  weak indirect α1-adrenoceptor agonist in addition to potent β-adrenoceptor antagonist.

 

Novocaine is mainly a sodium channel blocker and is used as an anesthesia; Corinfar ( nifedipine) and verapamil are both calcium channel blockers and can be used in this case ( hypertension coupled with hyperthyroidism) only when beta blockers are contraindicated.

54. A   patient  has   been   provisionally   diagnosed with pheochromocytoma at the stage of intermission. BP  is within  norm,  there  is a tendency towards  tachycardia. No urine pathologies. The  decision  has been  made  to perform a  provocative  test  with  histamine. What  drug  should  be kept  close at hand  for emergency aid in case of positive test result?

Explanation

 Pheochromocytoma is a tumor that affects the chromaffin cells of the adrenal medulla. It leads to the increased production of catecholamines ( epinephrine and norepinephrine) which have vasoconstrictive effects leading to hypertension. Amongst the listed drugs, Phentolamine which is a non-selective alpha adrenergic receptor blocker is used in the management of hypertensive emergencies majorly due to pheochromocytoma. This drug has a vasodilating  effect. 

 

Pipolphen ( promethazine) is an antihistamine and is used in the treatment of allergic reactions, motion sickness, anaphylaxis etc. Mesaton is a decongestant and a vasopressor.

55. A 54-year-old patient complains of weakness,  weight loss despite  the unchanged appetite, frequent urination, skin itch for six months. Some time ago the patient underwent treatment for furunculosis.  She has not been examined   recently.   Objectively:   malnutrition, dry skin with scratch marks. Small lymph nodes can be palpated in the axillary regions. Changes  in  the  internal   organs  are  absent. What   test  must  be  performed  in  the  first place?

Explanation

 

Notice that this patient shows signs related to metabolic changes in the body; these signs include weakness, weight loss, frequent urination amongst others. In order to rule out certain disorders such as diabetes, cushing syndrome, pheochromocytoma etc it is necessary to check the fasting glucose level.

56. A child is 1 month  10 days old. ”Gushing” vomiting has been observed  since the age of 3 weeks. The vomit volume exceeds the volume of the previous  feeding.  Objectively:  the chi- ld  is inert.  Skin  elasticity  and  tissue  turgor are  decreased. Hour-glass   deformity   sign  is positive.  The preliminary diagnosis  is pyloric stenosis.  What   treatment tactics  should   be chosen?

Explanation

 

Pyloric stenosis is a condition that occurs in babies within the first 6 months. In this case, the pyloric muscle in the stomach becomes enlarged thereby preventing the movement of food from the stomach into the small intestine. Note that in Pyloric stenosis, ‘. The vomit volume exceeds the volume of the previous feeding’ while in pylorospasm, volume does not exceed previous food content. Pyloric stenosis is typically managed with surgery; surgical pyloromyotomy known as Ramstedt's procedure, can also be done laparoscopically. Metoclopramide is an antiemetic drug and can be used to control vomiting.

57. A  46-year-old  woman  complains  of pain attacks  in the right lumbar  area, which irradiate   to   the   lower   abdomen,  and   nausea. This  kind  of  pain  attacks   has  never   been detected  in  the   patient  before.   Survey  X- ray of the abdominal cavity reveals no pathologic shadows.  Ultrasonic scan  detects a hyperechogenic growth  1,5 cm in diameter, which reflects sound wave, in the enlarged right renal pelvis. What diagnosis is most likely?

Explanation

Echogenicity of the tissue refers to the ability to reflect or transmit Ultrasound waves waves. Based on echogenicity, a structure can be characterized as hyperechoic (white on the screen), hypoechoic (gray on the screen) and anechoic (black on the screen). Stones Calculi are examples of hyperechoic materials and can be responsible for the blockage of the urinary tract. Tumors are hypoechoic.
58. Several  hours  before,  a 28-year-old  patient  suddenly  developed acute  headache and repeated vomiting,  then  lost  consciousness. Objectively:    focal   neurological   symptoms were not found. Pronounced meningeal symptoms   were   revealed.  BP-  120/80  mm Hg.  According  to  clinical  and  liquorological  findings  the  patient  was  diagnosed  with  subarachnoid haemorrhage.  After   administration of dehydrants the  patient’s condition  somewhat  improved. What  is the  main component of further emergency  care?

Explanation

 

This patient is experiencing a subarachnoid hemorrhage ( bleeding into the subarachnoid space). From the list of drug groups, anticoagulants ( against coagulation), antiaggregants (against aggregation) and fibrinolytics ( breakdown fibrin) will worsen the current situation if administered. A coagulant will help in the management of the hemorrhagic  situation.

59. A 24-year-old man has developed increasing headaches, vertigo, diplopia,  paresis of the facial muscles on the right, choking during swallowing.  The  signs  appeared on  the  5th day of respiratory disorder.  He was diagnosed with  acute  viral encephalitis. Determine the main direction of emergency  therapy:

Explanation

Acute viral encephalitis occurs from the invasion and further replication of a virus in the central nervous system. From the drugs listed Zovirax ( acyclovir) is an antiviral drug indicated in diseases such as herpes simplex virus, shingles , chicken pox , epstein barr virus etc. Ceftriaxone is a cephalosporin ( beta lactam) and used for bacterial infections. Lasix or furosemide is a loop diuretic while glucocorticoids have immunosuppressive, antiinflammatory and antipyretic effects.
60. A  23-year-old  woman,  who  works  as  a milk  and  dairy  inspector,  after  the  miscarriage  suffers  from  high  fever  up  to  38, 6o C , recurring chills, excessive  sweating.  Objectively: polyadenitis, pain in the lumbosacral spine,  swollen  left  knee  joint,  enlarged liver and spleen. What diagnosis is most likely?

Explanation

From anamnesis, we find out that the patient works as a milk and dairy inspector, she has a high fever with recurring chills ( this phrase is used in describing an ‘Undulant fever’). Recall that Brucellosis is a disease caused by the infection from   brucella spp.; a gram negative bacteria which can be gotten from unpasteurized milk and meat ( dairy products). treatment usually requires 2 antibiotics—doxycycline or trimethoprim/sulfamethoxazole plus gentamicin, streptomycin, or rifampin. Sepsis is a medical emergency caused by the body's reaction to an infection. fever, chills, excess sweating are also seen in patient undergoing this process but the key differential in this question is the profession (work ) of this patient ( milk/dairy inspector)
61. A 16-year-old  adolescent has been  hospitalized with complaints of unceasing nasal hemorrhage and  unbearable pain  in  his  right  elbow  joint.  Objectively:  the  large  joint is  enlarged  and  defigurated,  the  skin  over the joint is hyperemic. Arthropathy signs can be  observed  in the  other  joints.  Ps- 90/min. Blood test: erythrocytes - 3, 9 · 1012 /l, Нb- 130 g/l, color  index  - 1,0, leukocytes  - 5, 6 · 109 /l, platelets  -  220  ·  109 /l,  ESR-   6  mm/hour. Lee-White coagulation time:  start-  24\\\\\\\\\\\\\\\' , end- 27\\\\\\\\\\\\\\\' 10\\\\\\\\\\\\\\\'\\\\\\\\\\\\\\\'  . What  drug would be most efficient in the treatment of this patient?

Explanation

Cryoprecipitate is also known as cryoprecipitated antihemophilic factor. It is prepared from plasma and contains fibrinogen, von Willebrand factor, factor VIII, factor XIII and fibronectin. Used to prevent or control bleeding in people whose own blood does not clot properly. This includes patients with serious but rare hereditary conditions such as Hemophilia A (who lack factor VIII) and von Willebrand disease (who lack von Willebrand factor.
62.

A 67-year-old  man  complains  of dyspnea on exertion,  attacks  of retrosternal pain,  dizziness. He has no history of rheumatism. Objectively: pale skin, acrocyanosis. There are crackles  in the  lower  lungs. There  is systolic thrill  in  the  II  intercostal space  on  the  right, coarse systolic murmur conducted to the vessels of neck. BP- 130/90 mm Hg, heart rate - 90/min., regular  rhythm.  The liver extends  5 cm from  under  the  edge  of costal  arch,  shin edemas   are  present. Specify  the  suspected valvular defect:

Explanation

Aortic Stenosis: ( most common valvular disease). Causes: Idiopathic, Rheumatic heart disease. 

Characteristic Murmur: Heard at aortic area (2nd intercostal space, right sternal border) Ejection-systolic, high pitched murmur with ‘ Crescendo-decrescendo’ character. Radiates to the carotids, often presents with slow rising pulse and narrow pulse pressure. Patients may complain of exertional syncope

 

N/B Aortic stenosis causes Left ventricular hypertrophy

Mitral Regurgitation: Causes: Age related, Ischemic Heart disease, infective endocarditis, Rheumatic diseases,connective tissue disorders eg. Marfan’s syndrome, Ehler Danlos’ syndrome. 

Characteristic Murmur: Heard at Mitral area ( 5th intercostal space, midclavicular line), A Pansystolic, high pitched, ‘whistling’ murmur’ which radiates to the left axilla. A 3rd heart sound might be heard.

N/B Mitral Regurgitation causes left atrial dilatation. 

Note: Pansystolic murmurs can also be heard in Tricuspid regurgitation (5th intercostal space, left sternal border) and Ventricular Septal defect in children (at the left lower sternal border). 

63.

A  24-year-old   female   teacher  complains  of  dizziness  and   heart   pain   irradiating to   the   left   nipple.   Pain   is  not   associated with  physical  activity   and   cannot   be  relieved  by  nitroglycerin,  it  abates   after   taking  Valocordin and  lasts  an  hour  or  more. The  patient has  a  nearly  2-year  history  of this  disease.   Objectively:   Ps-  76/min.,  BP- 110/70 mm  Hg.  Heart  borders are  normal, heart sounds are clear. The ECG  shows respiratory  arrhythmia. Radiograph of the  cervicothoracic  spine reveals no pathology. Lungs, abdomen are unremarkable. What changes in blood formula can be expected?

Explanation

From the description of the above patient, we can deduce that the issue is more psychological/mental than physical. This can be proved by the fact that all diagnostic procedures are without any pathological changes. Note that the problem was resolved when the patient was prescribed  Valocordin (corvalol) a tranquilizer indicated for neuroses. Since it is a mental issue, the blood formula will show no changes.
64. A 51-year-old female patient complains of frequent defecation and liquid blood-streaked stools with mucus admixtures, diffuse pain in the  inferolateral abdomen, 6 kg weight  loss within the previous  month.  Objectively:  body temperature - 37, 4o C , malnutrition, skin  is pale and dry. Abdomen is soft, sigmoid is painful and spasmodic, makes a rumbling  sound. Liver  is dense,  painful,  extends  3 cm below the costal margin. What  is the most likely diagnosis?

Explanation

 Ulcerative colitis involves the continuous inflammation of the mucosa and submucosa of the colon that always involves the rectum. Characterised by the presence of crypt abscess and ulcers, bleeding; patients always present with bloody diarrhea ‘ liquid blood-streaked stools’.

sprue (Tropical and celiac) is a syndrome characterized by acute or chronic diarrhea, weight loss, and malabsorption of nutrients. The celiac form is due to the body’s immune reaction towards a gluten containing diet while the tropical form mostly leads to a decrease in the absorption of vitamins B9 and B12.

 

Bacillary dysentery is associated with the infection from Enterobacteriaceae; it is most times known as the severe form of shigellosis.

65.

Chief  physician  of a polyclinic charged  district  doctor  with  a task  tdetermine the pathological prevalence of  disease  N  ihis district.  What  document allowto  estimate the disease prevalence in the population of a medical district?

Explanation

66.

A 32-year-old woman complains  of dizziness,  headache,  palpitation,  tremor.  For the last several months she has been under outpatient  observation for  increased   arterial pressure. Since recently  such attacks  have become  more  frequent and  severe.  Objectively: the  skin is covered  with clammy sweat, tremor  of  the  extremities  is  present.  HR- 110/min., BP- 220/140 mm Hg. Heart sounds are  muffled.  Blood  test  results:  WBCs-  9, 8 · 109 /l, ESR-  22 mm/hour.  Blood  glucose - 9,8 millimole/l.  What  disease  is the  most  likely cause of this crisis?

Explanation

 

The woman not only shows signs of hypertension but also presents with changes relating to body metabolic activities ( tremors, tachycardia, dizziness, palpitations, sweating - these are signs relating to stress ( caused by the release of stress hormones ie, catecholamines). Pheochromocytoma is a tumor of the adrenal gland, it is characterised by the increased production of catecholamines ( adrenaline, noradrenaline) coupled with stress symptoms.

Essential hypertension is also called primary hypertension- a form of hypertension with no underlying disease as its cause. Preeclampsia is a pregnancy-associated hypertension coupled with other signs. Primary hyperaldosteronism  or Conn’s disease is also associated with the above stated signs , its other signs may include kidney related problems, excessive urination, low potassium content, flank pain etc.

67. A  45-year-old  patient complains  of pain in the epigastric region, left subcostal area, abdominal distension,  diarrhea, loss of weight. He has been suffering from this condition for 5 years. Objectively:  tongue  is moist with white coating near the root; deep palpation of abdomen reveals  slight pain in the epigastric region and Мауо-Robson’s point. Liver is painless and protrudes 1 cm from the costal arch. Spleen cannot  be palpated. What disease can be primarily suspected?

Explanation

 

It is stated that the patient has had this condition for 5 years; this rules out the possibility of an Acute form of the disease. The presence of an epigastric pain also confirms the answer isn't a Chronic enteritis. Pain and tenderness in the Mayo robson’s point indicates a pancreatic disorder. N/B Mayo-Robson's point – a point on the border of inner 2/3 with the external 1/3 of the line that represents the bisection of the left upper abdominal quadrant. This is where the tail of the pancreas is located. 

68.

A 58-year-old woman complains of spontaneous bruises,  weakness,  bleeding gums, dizziness. Objectively: the mucous membranes and skin are pale with numerous hemorrhages of various time of origin. Lymph nodes are not enlarged. Heart rate - 100/min., BP-  110/70 mm  Hg.  There   are  no  changes of internal organs.  Blood  test  results:  RBC- 3, 0 · 1012 /l,  Нb-  92 g/l, colour  index  - 0,9, anisocytosis,  poikilocytosis,  WBC - 10 · 109 /l, eosinophiles - 2%,  stab  neutrophiles - 12%, segmented neutrophiles - 68%,  lymphocytes - 11%,  monocytes  - 7%,  ESR-  12 mm/hour. What   index   should   be  determined  additionally by a laboratory to make a diagnosis?

Explanation

 

Since the patient complains about spontaneous bruises, bleeding gums, numerous hemorrhages of various time of origin, the blood platelet level should be checked. Platelets/ thrombocytes help in blood clot formation.

69. A 49-year-old patient complains of swallowing   disorder  that   intensifies   during eating solid food, hiccups, hoarse voice, nausea,  regurgitation, significant  weight  loss (15 kg within 2,5 months). Objectively:  body weight  is reduced;  the  skin  is pale  and  dry; vesicular  respiration; heart  sounds  are  sufficiently  sonorous;  heart  rate  is rhythmic.  The abdomen is soft,  no  pain  on  palpation. The liver  is  not  enlarged. What  investigation  is most  necessary  for  making  the  diagnosis  in this case?

Explanation

 

This patient is experiencing a swallowing disorder; from anamnesis and physical examination. We suspect that the problem lies along the GIT especially within the esophagus.To get a better view of this pathology, an Esophagoduodenoscopy should be carried out. This method involves the use of an endoscope, which is inserted via the mouth down to the duodenum for better visualization of the GIT.

70.

A 9-month-old child presents with fever, cough,   dyspnea.   The   symptoms   appeared 5  days  ago  after   a  contact   with  a  person suffering  from  URTI. Objectively:  the  child is in grave  condition. Temperature of 38o C , cyanosis of nasolabial  triangle  is present. Respiration rate  - 54/min., nasal flaring during breathing. There  was percussion  dullness on the right below the scapula angle, and tympanic sound over the rest of lungs. Auscultation revealed bilateral fine moist crackles  predominating on the right. What  is the most likely diagnosis?

Explanation

The presence of percussion dullness on the right just below the scapula accompanied by the result from auscultation (presence of fine moist crackles) are common signs found in cases of pneumonia. Pneumonia is an inflammation of the pulmonary tissue that occurs as a result of an infection e.g. bacterial, viral etc. laryngotracheitis is also termed Croup; it involves an infection of the larynx, trachea and lungs
71. A 50-year-old  man, who works as a polisher at a combine-building factory, addressed the  factory’s  sectorial  doctor  with  complaints of general fatigue, sensations  of numbness and  pain  in his fingers. Objectively:  the  skin of his fingers is pale. Reaction to pain, tactile and thermal stimuli was revealed to be slightly disrupted. No disruptions can be observed within the other organs and systems. What disorder is most likely?

Explanation

Pneumatic hammer disease is also known as  white fingers, traumatic vasospastic syndrome, vasospastic disease of the hands or hypothenar hammer syndrome, It occurs in individuals whose occupation requires the use of huge percussion machines such as hammers, drilling equipment  and also in sport related activities such as weightlifting, volleyball. This disorder is as a result disturbed blood flow to the affected hand.

 

Raynaud phenomenon possess similar physical changes but  usually occurs due to temperature changes or emotional distortions.

72.

A 27-year-old  womacomplains  of bleeding  gumsnasal  hemorrhages, multiple hematomas on the  skin of her  limbs and  on the  front  of hetorso,  extreme general  fatigue.  Blood  test:  Hb-  64  g/l,  erythrocytes - 2, 5 · 1012 /l,  reticulocytes -  16%,  platelets - 30 · 109 /l, ESR-  22 mm/hour.  What  approach would be most efficient for treatment of this pathology?

Explanation

73. A  23-year-old  patient had  taken  1 g of aspirin  to  treat   acute  respiratory infection. After  that  he developed an asthmatic  fit with  labored  expiration that  was  arrested by introduction of aminophylline. The  patient’s medical history is not burdened with allergies. The  patient has undergone two surgeries  for nasal polyposis in the past. What  diagnosis is most likely?

Explanation

 

Note that the patient has no history of allergy, no infection or strenuous activity were mentioned; these rule out options B,C and D respectively. Aspirin induced asthma presents with three Keys; An Asthmatic reaction, intake of aspirin  or a NSAIDs and presence of nasal polyps (all of which are present in the above case). The disorder is thought to be caused by an anomaly in the arachidonic acid metabolizing cascade. When medications such as NSAIDs or aspirin block the COX-1 enzyme, production of thromboxane and some anti-inflammatory prostaglandins is decreased, and in patients with aspirin-induced asthma, this results in the overproduction of pro-inflammatory leukotrienes ( especially cysteinyl leukotrienes), which can cause severe exacerbations of asthma and allergy-like symptoms.

74. A 58-year-old woman undergoing chemotherapy for her oncologic  disorder has developed sore throat. Examination revealed necrotic  areas  on the mucosa  of the pharynx and tonsils. Many of her teeth are afflicted with caries. In blood: neutrophilic granulocytes are practically  absent  against the background of leukopenia. Leukocytes are represented mainly by lymphocytes  and monocytes.  What disease can be suspected in the given case?

Explanation

 

Acute tonsillitis is an inflammation of the tonsils that frequently occurs in combination with an inflammation of the pharynx (tonsillopharyngitis). It is particularly common in children and young adults and is primarily caused by viruses and group A streptococci (GAS). Acute tonsillitis is characterized by the sudden onset of fever, sore throat, and painful swallowing. Tender, swollen cervical lymph nodes and tonsillar exudates may occur. Recall that white blood cells are classified into granulocytes ( eosinophils, basophils, neutrophils) and agranulocytes ( lymphocytes and monocytes); from analysis ‘ neutrophilic granulocytes ( granulocytes) are practically absent’ while lymphocytes and monocytes ( agranulocyte) are present. From this perspective, we can say this individual has Agranulocitar tonsillitis ( because agranulocytes are mainly represented).

St. Vincent\'s tonsillitis is an ulcero-membranous pharyngitis and tonsillitis caused by Fusospirochetal infection of the pharynx and palatine tonsils. It is characterised by; Superficial ulceration and necrosis of the tonsils and pharynx that often results in formation of a pseudomembrane, Foul smelling breath, Odynophagia ( dysphagia), Submandibular ,lymphadenopathy,Exudate, Patients typically have poor oral hygiene.

75. A   patient  complain of  constan dull pain  ithe  perineum and  suprapubic area, weak  flow  of  urine frequendifficult  painful   urination,  nocturia.  Th patient  has been suffering from this condition for several months,  during  which urination was becoming increasingldifficult, and pain in the perineum  has  developedOn  rectal  examination: thprostate is enlarged (mainly  its right lobe),  denseasymmetrical, central  fissure is smoothed out, the right lobe is of stony density, painless, tuberous. What disease is it?

Explanation

76.

A  child  is  10  years  old.  The  weight  is 46  kg.  Since  birth  the  child  has  been  gaining  excessive  weight.  The  parents are  full-bodied.  The child has undergone the following   tests:   carbohydrate  tolerance,  level   of 17-ketosteroids,  blood   electrolytes,  US   of adrenal glands, cranium  X-ray. The tests revealed no pathologies.The diagnosis of exogenic constitutive obesity  has been  made. What  direction of therapy should  be prioritized?

Explanation

 

Exogenic ( external cause) obesity is a form of obesity caused by the consumption of food more than one's capacity. Notice that all examinations done on the child were found to have no pathologies ( carbohydrate tolerance, level of 17-ketosteroids, blood electrolytes etc ) this rules out the possibility of an underlying cause of the obesity such as conn’s disease, type 2 diabetes etc. to correct this disorder, diet changes should be made, patient should also undergo regular exercise to lose the weight already gained.

77. An  8-year-old  child with a 3-year  history of diabetes was hospitalized in hyperglycemic coma. Specify the initial dose of insulin to be administered:

Explanation

 

Initial intravenous administration of 10 to 14 units of short-acting insulin has to be prescribed for the patient during the first hour. Continuous intravenous infusion of insulin in a dose 0,1 unit/kg/hour in 0,9 % sodium chloride infusion has to be given after that.

78. A  worker,  who  was  involved  in  fire  fighting  inside  the  building  that  stored  2 kg of mercury,  has been  delivered to a hospital with complaints  of emotional expansiveness, palpitations, excessive sweating, body tremor, heart pain. Within one day his condition aggravated. Objectively:  the  skin is pale  and moist.  The  patient is depressed. Permanent red  dermographism, erethism,  unstable  BP are observed.  What  drug is the serum  in this case?

Explanation

From anamnesis, we can tell this patient has had prior exposure to contents of mercury, this can  be confirmed by the presented symptoms which indicate poisoning by mercury. For the management of mercury poisoning, Unithiol should be administered; it is a chelating agent that is used as a remedy for poisonings by heavy metals. Other chelating agent include  dimercaprol, d-penicillamine etc

Atropine sulphate is used as a remedy for organophosphate poisoning, calcium tetacine is used in lead poisoning, amyl nitrite is used for cyanide poisoning.

79. During  meat  testing  Trichinella  was detected in diaphragm crura in one of the two muscular  tissue samples.  What  tactics should a doctor  choose regarding this meat?

Explanation

This meat has been found to be infested by trichinella; if consumed, this individual is likely to get trichinellosis. In this case, preservation of the meat by salting, boiling or freezing isn't going to salvage the meat. This meat should be properly disposed of to avoid consumption.
80. A 40-year-old patient was bitten by a stray dog an hour  ago. On  the  left shin there  is a bite  mark  - the  wound  is 4x2x0,5 cm in size. What  surgical aid would be most  efficient in this case?

Explanation

For first aid after a dog bite,  If the skin wasn’t broken, wash the area with warm water and soap. You can also apply an antibacterial lotion to the area as a precaution.

If the skin is broken, wash the area with warm soap and water and gently press on the wound to promote a small amount of bleeding. This will help flush out germs.

 

If the bite is already bleeding, apply a clean cloth to the wound and gently press down to stop the flow. Follow up with an application of antibacterial lotion and cover with a sterile bandage.

81. A 30-year-old  patient has been  hospitalized  with  diagnosis  of intestinal obstruction. The  surgery  revealed the  obstruction of the small  intestine   to  be  caused  by  a  helminth ball. What kind of helminth is it?

Explanation

 

Ascaris lumbricoides is one of the major causes of intestinal obstruction. Ascaris is a round intestinal worm infection transmitted by accidental ingestion of eggs present in human feces that contaminate food, soil, and water in areas of poor hygiene. This obstruction mostly occurs in the ileocaecal valve  due to the accumulation of these helminths. Symptoms include abdominal colic, constipation etc

82. Mother of a newborn  suffers from chronic pyelonephritis; she had a case of URTI before the  delivery.  The  delivery  is at  term,  for  a long  period   waters  were  not  breaking. On the  2nd  day  of  life  the  child  developed an erythematic  rash,  later   the  rash  developed into  blisters  approximately 1 cm in diameter filled with serous purulent substance.  Nikolsky’s  symptom  is positive.  After  the  blisters had been  lanced,  erosions  developed in their place. The child is inert. The child’s body temperature is subfebrile.  What  diagnosis  is most likely?

Explanation

 N/B Nikolsky sign is a skin finding in which the top layers of the skin slip away from the lower layers when rubbed. From anamnesis, the mother suffers from chronic pyelonephritis and a case of upper urinary tract infection. This is most likely the source of the newborn’s infection. Impetigo neonatorum is a dermatologically related bacterial infection. It is mostly caused by a staphylococcal aureus or  in lesser cases a streptococcus pyogenes  The bullous form is mostly seen in children under 2 years while the non bullous form is observed in older persons. Skin elements in the bullous form are characterised by the presence of fluid or pus-filled blisters surrounded by erythemic skin.

pseudofurunculosis, is observed in neonates and infants. The disease begins with the appearance of superficial pustules in the ostium of sweat glands (periporitis). Caused by staphylococcus aureus, which penetrates in the depth of the sweat gland and causes the forming of deep indurated painful nodules. The nodules are very similar to furuncles, but they have not got core in the center (hence the name pseudofurunculosis).

Vesiculopustulosis is a widespread purulent disease, which appears in the first years of life. In the ostium of the sweat glands numerous pustules appear, filled with white yellow matter, the size of a pin head to a small pea, they do not merge with each other and are surrounded by bright edematous circles.

83. Three  weeks after  a case of acute tonsillitis the  patient is still weak,  inert,  subfebrile, his retromaxillary lymph nodes  are enlarged. Tonsils are flabby, stick together with arches, there  are  purulent plugs in lacunae.  What  is the most probable diagnosis?

Explanation

 

Tonsillitis is considered chronic if it persists for more than 2 weeks- this rules out the option of Acute lacunar tonsillitis ( which is the inflammation of the mucous lining surrounding the tonsillar crypts). The question goes on to describe an inflamed tonsil. In pharyngitis, the patient will present with difficulty in swallowing.

84.

The  following  indicators were  calculated to  analyse  population health  and  treatment quality in a cardiological hospital: primary cardiovascular morbidity - 62%;  total  cardiovascular morbidity - 483,55%; cardiovascular mortality - 10,9%; proportion of cardiovascular  mortality   within  total  mortality   - 67,0%; primary  disablement caused  by cardiovascular  diseases - 16,2 per 10.000 population. What indicator is an extensive value?

Explanation

The extensive statistical indicator is used to determine a structure of a disease ( in this case, how many percent ( the proportion) of the total diseased fall under cardiovascular cases) ie., it shows, what part from the general number of all diseases is made with this or that disease which enters into total.

The intensive parameter characterizes frequency or distribution. It shows how frequently the given phenomenon occurs in the given environment.

85. A 6-year-old  child complains  of frequent liquid  stool  and  vomiting.  On  the  2nd  day of disease  the child presented with inertness, temperature rise up to 38, 2o C , Ps- 150/min., scaphoid abdomen, palpatory painful sigmoid colon, defecation 10 times  a day with liquid, scarce stool with mucus and streaks  of green. What is the provisional  diagnosis?

Explanation

Bacterial gastroenteritis is caused by a variety of organisms, including Campylobacter, Salmonella, Shigella, Yersinia, Vibrio cholerae, Staphylococcus aureus, diarrheagenic Escherichia coli, Clostridium difficile, Clostridium perfringens, and non cholera Vibrio species. Shigellosis is also known as bacillary dysentery caused by the infection of the shiga toxin. It is characterised by High-fever, tenesmus,Profuse inflammatory, mucoid-bloody diarrhea and basically affects the large intestine. This patient experiences pain in the sigmoid colon accompanied by profuse diarrhea ( mixed with mucus)
86. The  institutions that  take  part  in  medical examinations include prevention and treatment facilities,  medical  board  of  Ministry of Defense, medical board  of Ministry of Home Affairs, medico-social expert commissions, forensic medical boards etc. What institutions  are responsible for temporary disability examination?

Explanation

  The purpose of the  medicosocial expertise is for definition of the fact, degree, reason of a temporary and permanent disability and the necessity of release from causative activity . This kind of expertise implements preventive establishments at temporary disability and medicosocial commissions of experts (MSCE) - at permanent disability.
87. After   a  lengthy   march   an  army   regiment   has   set   camp   for   3   days   near   a settlement. Sanitary-hygienic investigation detected several  water  sources.  Choose  the source  that   would  satisfy  the  demands   for potable water  the  most  under  the  given  field conditions:

Explanation

 

Artesian well is a type of well from which water flows under natural pressure without pumping. It is dug or drilled wherever a gently dipping, permeable rock layer (such as sandstone) receives water along its outcrop at a level higher than the level of the surface of the ground at the well site. From the above listed options, the artesian well is the best source of clean water.

88. Clinical statistical  investigation was performed to  determine efficiency of a new pharmacological preparation for patients with  ischemic  heart  disease.  What  parametric test (coefficient)  can be used to estimate reliability of the results?

Explanation

The T-test is a statistical, hypothetical test used to compare the mean (average) of two related variables. For example comparing the mean of the experimental group to that of the controlled group. It lets you know how significant the differences are.

 

The Z-test also compares the means of two populations although it relies on the Variance ( the square root of the mean deviation).

89. A 30-year-old parturient woman was delivered  to  a maternity hospital  witfull-term pregnancy She  complain of  severe   lancinating  paiin the  uterus  that  started 1 hour ago, nausea, vomiting, cold sweat. Anamnesis states  cesarea section  2  years  ago.  Uterine contractions stopped. Skin and mucous membranes arpale.  Heart rate  is 100/min., BP is 90/60 mm Hg. Uterus has no clear marginsis sharply  painful.  No  heartbeat can  be auscultated in the fetus. Moderate bloody discharge from the uterus can be observed. Uterus cervix is 4 cm open. Presenting part is not visible. The most likely diagnosis is:

Explanation

90. A  parturient  woma is  23  years   old. Internal obstetric examination shows the uterine cervito  be  completely  open.  Fetal bladder iabsent Cephali presentation is observe in  the  plane   of  the  small  pelvic outlet Sagitta sutur is  at  the   longitudinal  section  of the  small  pelvioutlet,  small fontanel  is situated closer to the uterusWhat cephalic position  will the newborn  have during birth in this case?

Explanation

91. During  the dynamic observation of a parturient woman in the second stage of labor it was registered that the fetal heart  rate decreased to 90-100/min. and did not normalize after contractions. Vaginal examination revealed the complete cervical dilatation, the fetahead  filling thentire  posterior surface of  the  pubic  symphysis  and  sacral  hollow; the  sagittal  suture  was in the  anteroposterior diameter of the pelvic outlet,  the posterior fontanelle was in fronunder  the pubic arch. What  plan  for  further labour   management should be recommended?

Explanation

92. An 8-year-old girl periodically  has sudden short-term heart  pains, sensation of chest compression, epigastripain, dizziness, vomiting.  Objectively the  patient ipale,  respiratory  rate  - 40/min., jugular pulse is present. Ps-  185/min.,   of  poo volume BP 75/40 mm Hg. ECG  taken  durinan attack  shows ectopic P waves, QRS wave is not deformed. At the end of an attack a compensatory pause is  observedThe  most  likely  cause  of  the attacis:

Explanation

93.

A  16-year-old   girl  has  primary amenorrhea, no pubic  hair  growth,  normally developed mammary glands; her genotype is 46 ХY; uterus  and vagina are absent.  What  is your diagnosis?

Explanation

Testicular feminization syndrome is commonly called complete androgen insensitivity syndrome. This is a genetic condition in which a male (XY) fetus is unresponsive to male hormones ( androgens). This occurs mostly due to defects in androgen receptor resulting in a normal appearing female ( 46, XY DSD)  ( DSD- differences in sex development). The individual will present with female external genitalia with scant axillary and pubic hair, rudimentary vagina, uterus and fallopian tubes are absent due to the persistence of the anti-mullerian hormone from the testes. 

 

Mayer-Rokitansky-Kuster-Hauser syndrome is also known as Mullerian Agenesis; it presents with primary amenorrhea due to lack of uterine development in females with fully developed secondary sexual characteristics ( functional ovaries).

94. A planner designs a heating  system for a pre-school educational establishment. The highest air temperature should be in the following room:

Explanation

95.

A 27-year-old sexually active woman complains  of numerous vesicles on the  right sex lip, itch and burning.  Eruptions regularly appear  before   menstruation and  disappear 8-10 days  later.  What  is the  most  likely  diagnosis?

Explanation

 Herpes simplex viruses are enveloped double stranded linear viruses. HSV-1 is also known as oral herpes ( gingivostomatitis, keratoconjunctivitis etc) while HSV-2 is known as genital or neonatal herpes. The clinical presentation of genital herpes include  pain, itching, dysuria, vaginal and urethral discharge, tender lymphadenopathy, appearance of herpes vesicles on the  external genitalia, labia majora, labia minora, vaginal vestibule - for women and glans penis, the prepuce, the shaft of the penis, and sometimes on the scrotum, thighs, and buttocks- for men.

Primary syphilis usually presents with a localized painless hard chancre on the genitals.

Cytomegalovirus is also known as human herpes virus-5, it is usually seen in immunocompromised patients and infected cells have characteristic ‘ owl's eye’ intranuclear inclusions.
96. A  35-year-old  woman  addressed a gynecological  in-patient department with complaints of regular pains in her lower abdomen, which increase  during  menstruation,  and  dark-brown  sticky  discharge   from the  genital  tracts.  On  bimanual  examination: the  uterine body is slightly enlarged, the appendages are  not  palpated. Mirror  examination   of  the  uterine cervix  reveals  bluish spots. What diagnosis is most likely?

Explanation

Endometriosis is usually a benign problem of endometrial tissue implantation outside the uterine cavity. Manifestations include pain, dyspareunia, painful defecation, sacral backache, hypermenorrhea, and infertility. In cervical endometriosis, implantation takes place in the cervix leading to changes in this area during examination
97. A 10-year-old patient has a history of mild bronchial asthma.  During  a regular  check-up the patient should be recommended:

Explanation

98.

A baby was born by a young smoker.  The labou was  complicateby  uterine inertia, difficult delivery of the babys head and shoulders The  babys  Apgar   score  was  4. Whicof the  followinis a risk factor  for a spinal cord injury?

 

Explanation

99. An  emergenc situatioat  a  chemical plan caused   acut occupational  intoxication. A doctor  who revealed the case of ”acute occupational disease (intoxication)” must notify the following authority:

Explanation

100. A 27-year-old woman has been treated in a  surgical  department for  pleural  empyema for  6 months.  Multiple  paracenteses of  the pleural cavity were performed along with anti- bacterial treatment. The patient’s condition is slowly aggravating;  attempts to  fully stretch the lung were unsuccessful. Choose  the tactics:

Explanation

Note that this condition is restricting the lungs from fully stretching and has resisted multiple antibacterial therapy. Decortication is a procedure that involves the surgical removal of a restrictive layer of fibrous tissue overlying the lung, chest wall, and/or diaphragm. The aim of decortication is to remove this layer and allow the lung to re expand.
101. A week ago a 65-year-old patient suffered an acute myocardial infarction, his general condition deteriorated: he complains of dyspnea at rest, pronounced weakness. Objectively: edema of the lower extremities, ascites is present. Heart borders are extended, paradoxical pulse is observed 2 cm from the apex beat to the left. What is the most likely diagnosis?

Explanation

 

An aneurysm refers to the bulging out of blood vessels; usually occurs in the aorta or vessels located at the base of the heart septum. In most cases, aneurysm occurs secondary to a myocardial infarction. When the heart muscle (cardiac muscle) partially dies during a heart attack, a layer of muscle may survive, and, being severely weakened, start to become an aneurysm. Blood may flow into the surrounding dead muscle and inflate the weakened flap of muscle into a bubble.

102. A 60-year-old patient complains of nearly permanent sensation of heaviness and fullness in the epigastrium, which increases after eating, foul-smelling eructation, occasional vomiting with food consumed 1-2 days ago, weight loss. 12 years ago he was first found to have an ulcer of pyloric channel. The patient has taken ranitidine for periodic hunger pains. The patient’s condition has been deteriorating over the last 3 months. Objectively: splashing sound in the epigastrium is present. What kind of complication is it?

Explanation

 

Pylorospasm is a condition in which the Pyloric sphincter fails to relax properly due to continuous contractions (spasms). The key finding here is that vomiting occurs a few minutes after eating and the volume does not exceed previous food content; this helps differentiate it from Pyloric stenosis which occurs due to the narrowing of the pyloric part of the stomach leading to improper digestion. Pyloric stenosis is characterized by splashing sound in the epigastrium, Projectile vomiting followed by an eagerness to take food. In esophageal achalasia, the patient will firstly experience Dysphagia ( difficulty in swallowing) followed by vomiting; this occurs due to failure in the relaxation of the lower esophageal sphincter and peristalsis.

103. A prematurely born girl is now 8 months old. She has dyspnea, tachycardia, hepatosplenomegaly, physical developmental lag, limb cyanosis. There is parasternal cardiac hump, auscultation revealed systolodiastolic murmur in the II intercostal space on the left. BP is 90/0 mm Hg. What disease can be suspected?

Explanation

Patent ductus arteriosus is a congenital heart anomaly in which the ductus arteriosus fails to close at birth. PDA is normal in utero and normally closes only after birth. Due to the failure of the ductus arteriosus ( a connection between the aorta and pulmonary artery), oxygenated blood flows from the aorta into the pulmonary artery then back to the lungs for more oxygenation. Key signs include: dyspnea, tachycardia, cyanosis of the lower extremities etc. On objective examination, we observe increased systolic pressure, left subclavicular thrill, a continuous rasping systo diastolic murmur in the 2nd intercostal space on the left; this murmur can also be termed like “a machine gun” or “rolling thunder”. Echocardiogram and normal chest x-ray are used in the diagnosis. One major finding on chest x-ray is the cardiac Silhouette (loss of normal borders between thoracic structures). 

 Tetralogy of Fallot is caused by the anterosuperior displacement of the infundibular septum and is the most common cause of cyanosis in early childhood. It consists of 4 anomalies: Ventricular septal defect, Right ventricular hypertrophy, pulmonary valve stenosis and a misplaced (overriding) aorta. A systolic ejection murmur is common in this anomaly.
104. Three days ago a boy underwent removal of a foreign body from under a nail plate. 2 days later he deeloped acute pulsating pain at the end of the nail bone which aggravated at pressing. Nail fold became hyperemic, body temperature rose up to 37, 5oC, there was a change in nail plate colour. What is the most likely diagnosis?

Explanation

105. A 25-year-old woman complains of menstruation retention lasting for 3 years. The patient explains it by a difficult childbirth complicated with profuse hemorrhage, weight loss, brittleness and loss of hair, loss of appetite, depression. Objective examination reveals no pathologic changes of uterus and uterine appendages. What pathogenesis is characteristic of this disorder?

Explanation

Gonadotropin ( luteinizing hormone and follicle stimulating hormone) are hormones that act on the gonads and stimulate the production of sperm ( in males) and ovaries ( in females). LH stimulates the Leydig cells of the testes and the theca cells of the ovaries to produce testosterone (and indirectly estradiol), whereas FSH stimulates the spermatogenic tissue of the testes and the granulosa cells of ovarian follicles, as well as stimulating production of estrogen by the ovaries. Follicle stimulating hormone stimulates the growth of the ovarian follicles thereby enhancing ovulation and menstruation. This patient has problems relating to menstruation and childbirth; a reduction in FSH is most likely the reason behind these changes.

 

Estrogen is the primary female sex hormone produced in the ovaries and a little from the placenta. It is responsible for puberty changes in females. Progesterone is also a sex hormone, produced from the corpus luteum in the ovaries, just like estrogen, it is involved in changes during pregnancy, menstruation, embryogenesis etc. Note that from examination, there were no pathologic changes of uterus and uterine appendages; this statement rules out the option of estrogen and progesterone ( if these hormones were responsible, there will be a change in uterine appendages eg in the ovaries).

106. A patient in a clinical death condition is being resuscitated through mouth-to-mouth artificial pulmonary ventilation and external cardiac massage. A doctor noticed that air does not flow into the patient’s airways and his head and torso are positioned at the same level. Why is artificial respiration not effective in the given case?

Explanation

107. A 45-year-old patient with urolithiasis had an attack of renal colic. What is the mechanism of the attack development?

Explanation

 

In the presence of kidney stones (urolithiasis), urine builds up ( accumulates) in the urinary tract leading to the dilation, stretching and spasm of these tracts. These corresponding changes lead to the development of painful sensation that radiates towards the groin.

108. A 26-year-old woman has attended maternity center complaining of her inability to become pregnant despite 3 years of regular sex life. Examination revealed the following: increased body weight; male-type pubic hair; excessive pilosis of thighs; ovaries are dense and enlarged; basal body temperature is monophasic. The most likely diagnosis is:

Explanation

Also called Stein- Leventhal syndrome, Ovarian sclerocytosis is the process of ovarian regeneration, accompanied by the formation of small cystic formations up to 1 cm in size. It usually occurs in patients with polycystic ovarian syndrome. Key findings include; infertility, male pattern hair distribution, weight loss, hormonal disbalance, bilateral enlargement of the ovaries, violation of menstrual cycle etc. 

Adrenogenital syndrome ( congenkital adrenal hyperplasia) is a condition characterized by the enlargement of the adrenal gland coupled with the excess production of androgens (sex hormones).

 

Premenstrual syndrome usually occurs just before a woman’s menses and is characterised by emotional, physical and behavioural changes.

109. The process of open-cut mining requires drilling and blasting operations, rock and ore excavation, transportation of ore to fragmentation and sorting factories and transportation of barren rock to slag-heaps, road building and maintenance, repair works. What factor of production is most important for miner’s health?

Explanation

 

Mining involves the extraction of materials/ minerals from the earth; just like the question states, it involves processes such as drilling, blasting, excavation, sorting etc. Miners are usually exposed to lots of hazards such as earth collapse, dust inhalation, noise, vibrations etc. Of all the above listed hazards, exposure to dust is the most important/frequent hazard to a miners health; depending on the type of dust, this can lead to restrictive lung diseases such as pneumoconiosis, silicosis etc.

110. A woman addressed a gynecologist on the 20th day of puerperal period with complaints of pain in the left mammary gland puruent discharge from the nipple. Objectively: Ps- 120/min., body temperature is 39oC. The left mammary gland is painful, larger than the right one, the skin there is hyperemic; in the upper quadrant there is an infiltrate 10x15 cm in size with soft center. Blood test: ESR50 mm/hour, leukocytes - 15, 0 · 109/l. What would be the treatment tactics?

Explanation

 

In the left mammary gland, we observe the presence of purulent discharge, and presence of a mass. From the above description, this woman should be  taken to the surgical department for the drainage of the purulent content and removal of the infiltrate/ mass. A conservative treatment will not be possible at this stage because of the present size of the infiltrate.

111. The correlation between the service record and eosinophil concentration in blood was studied in workers at dyeing shops of textile factories. What index will be most informative for the analysis of this data?

Explanation

Correlation factor/ coefficient is a statistical measure that shows the relationship between two variables; in the case between service records and eosinophil concentration. Most common types include the Pearson’s correlation coefficient and rank coefficient.
112. A 3-month-old girl has rhinitis, dyspnea, dry cough. She has been sick for 2 days. Objectively: pale skin, acrocyanosis, hypopnoe; breathing rate is 80/min.; over the whole pulmonary surface there is vesiculotympanitic (bandbox) resonance observed with numerous bubbling crackles. The most likely diagnosis is:

Explanation

 

Note that acute bronchiolitis and Acute bronchitis are very similar and basically present with almost the same symptoms. One key difference is the age range of people affected ; Bronchitis is found in individuals of all age ranges although mostly seen in older children and adults  while bronchiolitis is only found in younger children especially below 2 years. From analysis, we hear vesicular tympanic sound and bubbling rales / crackles - these can  be found in both diseases; but the key difference here is the age of the child (3 months).

113. A 24-year-old patient complains of gaining body mass and increased appetite. Objectively: built of hypersthenic type, body mass index is 33,2 kg/m2, waist circumference is 100 cm. Waist to hips circumference ratio is 0,95. What is the provisional diagnosis?

Explanation

Based on the body mass index, there are three classes of obesity;

 Class 1 (low-risk) obesity, if BMI is 30.0 to 34.9.

Class 2 (moderate-risk) obesity, if BMI is 35.0 to 39.9.

 

Class 3 (high-risk) obesity, if BMI is equal to or greater than 40.0.

114. A 36 year old female patient complains ofgeneral weakness, edemas of her face and hands, rapid fatigability during walking, diffi- cult diglutition, cardiac irregularities. These symptoms developed 11 days after holiday at the seaside. Objectively: face erythema, edema of shin muscles. Heart sounds are muffled, BP is 100/70 mm Hg. In blood: ASAT activity is 0,95 millimole/h·l, ALAT1,3 micromole/h·l, aldolase - 9,2 IU/l, creatine phosphokinase - 2,5 micromole Р/g·l. What method of study would be most specific?

Explanation

 

From the presented symptoms; we can clearly see that this patient has problems with activities regarding muscular action eg.difficulty swallowing, cardiac irregularities etc. From biochemical analysis, we observe an increased aldolase and Creatinine levels. Note that aldolase levels are usually increased in individuals with liver or muscular damage and creatinine levels are also increased in individuals with muscular pathologies; creatinine normal range- 0.5-1.5 mEq/dl. These signs signal the doctor that the problem lies within the muscle; and a muscle biopsy should be carried out for better diagnosis.

115. A 14-year-old girl has fainted during a meeting. The day before she complained of a headache. The skin is pale, the limbs are cold, shallow breathing, heart sounds are muffled; heart rate is 51/min.; BP is 90/50 mm Hg. The abdomen is soft. Meningeal symptoms are negative. Make the provisional diagnosis:

Explanation

116. For the last 15 years a 48-year-old patient has been working at the factory producing synthetic resins. Lately he has been complaining of significant general fatigue, headaches, frequent urination (predominantly during the day), red color of urine. What complication of benzene nitrocompounds poisoning can be suspected?

Explanation

Aromatic compounds such as benzene and its products (es nitrobenzene, benzidin etc) are major components in factories producing dyes and resins and upon exposure are dangerous  to health. One major negative effect is the conversion of hemoglobin to methemoglobin. In recent times, it has been proved that these compounds act as carcinogens and lead to bladder related tumors.

Chronic Prostatitis is ruled out since the patient wasn’t experiencing pain while passing out urine.

 

In Chronic Cystitis, the patient often observes severe burning sensation in the pelvic region.

117. A 60-year-old woman developed weakness, vertigo, rapid fatigability during the last year. Recently there have been dyspnea and paresthesia observed. Objectively: skin and mucous membranes are pale and icteric. Lingual papillae are smoothed out. Liver and spleen are situated at the edge of costal arch. Blood test: Hb- 70 g/l, erythrocytes - 1, 7·1012/l, blood color index - 1,2, macrocytes. What drug can be prescribed on pathogenetic grounds?

Explanation

Notice that from the blood analysis, Macrocytes are present- these refer to irregular large Rbcs. The erythrocyte level is also very low ( norm- Female: 3.5 − 5.5 · 1012/L) - this indicates an anemia . The major causes of macrocytic anemia include Vitamin b12 deficiency, Vitamin b9 deficiency or medications such as antiretroviral drugs. Symptoms include loss of appetite, brittle nails , pale skin, fatigue etc.

 

Ascorbic acid or vitamin C deficiency will lead to scurvy, petechial bleeding or bruises are common in this case.

118. In the morning upon waking a 65-yearold patient developed weakness in the rightside limbs, speech disorder, decreased sensitivity of the left side of the body. On examination: conscious, BP- 100/60 mm Hg, motor aphasia, right-sided central hemiparesis and hemihypalgesia. Make the preliminary diagnosis:

Explanation

Ischemic strokes occur when blood flow to the brain is blocked by a blood clot.

There are two major types of ischemic stroke:

Thrombotic strokes are caused when a blood clot forms in an artery leading to the brain.

Embolic strokes begin with a clot forming elsewhere in the body — such as the heart or neck — that breaks loose and travels to the brain.

Patients may experience a combination of symptoms that include numbness or weakness on one side of the body or face, trouble speaking and difficulty with vision or balance.

A hemorrhagic stroke happens when a weak blood vessel bursts and bleeds into the brain.

People who experience this type of stroke, in addition to other stroke symptoms, will likely experience a sudden onset headache or head pain — a warning sign that might not occur during ischemic stroke.

Patients may experience one of the following types:

 
  • Intracerebral hemorrhage, a weak blood vessel breaking inside the brain

  • A subarachnoid hemorrhage, a weak blood vessel breaking on the surface of the brain

119. The objective of a statistical study was to find out the extent of seeking medical care by the population. For this purpose 300 residents of the area were interviewed. Information was collected by means of a special questionnaire. What method of collecting information was used by researchers?

Explanation

 

Anamnestic assessment is a specific type of clinical assessment whereby the examiner attempts to identify violence risk factors through a detailed examination of the individual's history of violent and threatening behavior. This is a technique in patients clinical assessment.

120. A 57-year-old woman complains of having a sensation of esophageal compresion, palpitation, difficult breathing during eating solid food, occasional vomiting with a full mouth, ”wet pillow” sign at night for the last 6 months. Objectively: body tempearture - 39oC, height - 168 cm, weight - 72 kg, Ps76/min., BP- 120/80 mm Hg. X-ray revealed a considerable dilation of the esophagus and its constriction in the cardial part. What pathology is most likely to have caused dysphagia in this patient?

Explanation

Esophageal achalasia is an esophageal motility disorder involving the smooth muscle layer of the esophagus and the lower esophageal sphincter (LES). It is characterised by an increased LES tone, incomplete LES relaxation and lack of esophageal peristalsis. The sign of wet pillow results from increased salivation and nocturnal discharge of saliva and mucus from the mouth and is observed in esophageal pathologies. Diagnosis is confirmed by x-ray examination and from the above patient, we see the dilation of esophagus and its constriction in the cardial part which confirms our diagnosis.
121. A 25-year-old man complains of pain in the lower third of his left thigh, which occurs both with and without physical exertion. Possibility of trauma is denied by the patient. Objectively: skin colour is normal; pastosity and pain can be detected with deep palpation; knee joint mobility is reduced. Xray of distal femoral metaphysis shows an area of destruction and spicule. Blood test: immature cells, no signs of inflammation. The most likely diagnosis is:

Explanation

Osteosarcomas are primary malignant tumors of bone that are characterized by the production of osteoid or immature bone by the malignant cells. Characteristics of osteogenic sarcoma include; the presence of osteoid-producing cells ( malignant osteoblasts- immature cells), present as painful enlarging mass . A major sign seen on x-ray is the Codman triangle ( a subperiosteal lesion), in some literature, it is described as ‘resembling melting sugar’ or ‘sunburst pattern’.
122. A 30-year-old woman suffers from polycystic renal disease. She has been admitted with signs of fatigue, thirst and nocturia. Diuresis is up to 1800 ml per day. BP is 200/100 mm Hg. Blood test: erythrocytes - 1, 8 · 109/l, Hb- 68 g/l. Urine analysis: specific gravity - 1005, leukocytes - 50-60, erythrocytes - 3-5 in the vision field, creatinine - 0,82mmol/l, potassium - 6,5 mmol/l, glomerular filtration rate - 10 ml/min. What tactics would be leading in the patient’s treatment?

Explanation

 

Hemodialysis is a procedure used for removal of waste materials from the blood through filtration. It is carried out on people with renal failure ( in this patient, the GFR is 10ml/min ( about 90-120 ml/min). Notice that his blood values are not normal; K- 6.5mmol/L ( norm- 3.5-5.5), ,Creatinine - 0.8mmol/l glucose level is also high. Recall that the kidney has three basic functions; filtration, reabsorption and secretion, and a distortion in these values indicates a kidney related issue.

123. In a rural health care area there is a case of child dying during the first month of life. To analyse this situation, among other measures, an expert assessment of medical records is performed. What medical document should be considered first?

Explanation

124. A region attended by a central regional hospital demonstrates increased hemorrhagic stroke morbidity. Essential hypertension morbidity, however, remains at the same level and is below the average level registered within the larger area. What managerial decision should be made in this case?

Explanation

125. A 42-year-old man was delivered to a surgical in-patient department with complaints of icteric skin, pain in the right subcostal area. Biochemical blood analysis: total bilirubin - 140 mcmol/l, direct bilirubin - 112 mcmol/l. On US: choledoch duct - 1,4 cm, a concrement is detected in the distal area. Gallbladder is 40 cm, no concrements. What treatment tactics should be chosen?

Explanation

The above patient presents with signs of liver/ gallbladder pathology. From ultrasound results, we can conclude that a stone is present in the bile duct (distal part) and NOT the gallbladder. With this result, a cholecystectomy is ruled out and the preferred treatment tactics is Endoscopic papilla sphincterotomy which is a procedure used for the removal of bile duct stones.
126. 4 weeks after myocardial infarction a 56- year-old patient developed acute heart pain, pronounced dyspnea. Objectively: the patient’s condition is extremely grave, there is marked cyanosis of the face, swelling and throbbing of the neck veins, peripheral pulse is absent, carotid artery pulse is rhythmic, 130/min., BP is 60/20 mm Hg. Auscultation of  the heart reveals extremely muffled sounds, percussion reveals heart border extension in both directions. What is the optimal treatment tactis for this patient?

Explanation

We can pick out: Extended heart borders, low b.p and an increased pulse . These three signs are known as Beck’s triad. They indicate a Cardiac Tamponade ( the impaired pumping ability of the heart due to accumulation of fluid in the pericardium). Note that percussion reveals heart border extension in both directions. The most suitable approach  should be a pericardial puncture ( pericardiocentesis) to remove the fluid in the pericardium. A thoracotomy can be carried out to gain access to the pleural cavity.
127. A 45-year-old man was delivered to a hospital with complaints of vomiting with streaks of blood, loss of weight. On esophagofiberscopy a cauliflower-shaped mucosal growth was detected in the abdominal esophagus. The mucosa there bleeds on contact. What preliminary diagnosis can be made?

Explanation

128. A 35-year-old man complains of intense lumbar pain irradiating to the inguinal area, external genitalia, thigh; frequent urination, chill, nausea, vomiting. Objectively: positive Pasternatsky’s symptom (costovertebral angle tenderness). Urine analysis revealed that RBCs and WBCs covered the total field of microscope; the urine exhibited high protein concentration. These clinical presentations were most likely caused by the following pathology:

Explanation

129. A 20-year-old patient complains of pain in the left lumbar region, arterial pressure rise up to 160/110 mm Hg. US revealed that the structure and size of the right kidney were within age norms, there were signs of 3 degree hydronephrotic transformation of the left kidney. Doppler examination revealed an additional artery running to the lower pole of the kidney. Excretory urogram shows a narrowing in the region of ureteropelvic junction. Specify the treatment tactics:

Explanation

It is stated that there is a 3rd degree hydronephrotic change in the left kidney;  Hydronephrosis is swelling of one or both kidneys (dilation of the renal pelvis and calyces). Kidney swelling happens when urine can't drain from a kidney and builds up in the kidney as a result. There are three degrees of Hydronephrosis: 

 
  • 1st degree (mild)-   moderate renal pelvis dilation with few calyces

  • 2nd degree (moderate)- full renal pelvis dilation involving all calyces

  • 3rd degree (severe)- includes the above signs coupled with thinning of the parenchyma. In severe hydronephrosis, surgery is recommended.

130. As a result of load lifting a 68-year-old woman developed acute pain in the lumbar region, in a buttock, posterolateral surface of her right thigh, external surface of the right shin and dorsal surface of foot. Objectively: weakness of the anterior tibial muscle, long extensor muscle of the right toes, short extensor muscle of the right toes. Low Achilles reflex on the right. Positive Lasegue’s sign. What examination method would be most effective for specification of the diagnosis of L5 root discogenic compression?

Explanation

131. A 23-year-old female patient has been suffering from mental disorder since the age of 18, the course of disorder has no remission periods. At hospital the patient mostly presents with non-purposeful foolish excitation: she makes stereotypic grimaces, exposes herself, publically masturbating, loudly laughs, repeating stereotypical obscene shouts. The patient should be prescribed:

Explanation

 

Neuroleptics are antipsychotic drugs indicated for cases such as schizophrenia, bipolar disorders. They are used in managing psychosis and related symptoms such as delusions, hallucinations, paranoia or disordered thought. Tranquilizers are used in anxiety disorders, fear, tension etc. Nootropics are drugs used in boosting memory and creativity.

132. During preventive ultrasound scan of abdomen performed during regular checkup in a school the following was revealed in an 11-year-old student of the 5th grade: the left kidney is 3 cm below the normal position, its shape, size and structure are within the norm, the contralateral kidney cannot be observed at its proper place. The preliminary diagnosis is as follows: congenital anomaly of renal development, dystopic left kidney, right kidney is absent or pelvic dystopic. What Xray method would be required for making the final diagnosis and determining the functional capacity of both kidneys?

Explanation

133. A young woman suffering from seborrhea oleosa has numerous light-brown and white spots on the skin of her torso and shoulders. The spots have clear margins, branny desquamation, no itching. What provisional diagnosis can be made?

Explanation

The above description is typical for Tinea (pityriasis) versicolor which is caused by malassezia spp. A yeast like fungus. It is characterised by areas of hypopigmentation ‘white spots’. Hyperpigmentation can also occur due to inflammatory response - ‘areas of brown spots .’ It differs from dermatophytes because it is less pruritic (itchy).

 

Pityriasis rosea is a viral rash that resembles small oval red patches and are very itchy.

134. In a traffic accident two persons died. An appointed forensic medical expert was called on duty to another accident location; therefore, according to the crininal law in force in Ukraine, an investigator called in the following specialist for examination of the location and bodies:

Explanation

135. A patient is 28 years old. He has been suffering from mental disorder since he was 22. His current condition has changed acutely: for 3 days the patient has been refusing to leave his home. He claims that there is a ”telepathy” occurring between him and other people, through which he receives ”thoughts of strangers” and transmits his own thoughts for everyone to hear. He thinks his thoughts and actions are manipulated through this ”telepathy”. Make the preliminary diagnosis:

Explanation

Schizophrenia is a mental disorder characterised by disturbances in thought and verbal behaviour,perception, motor behaviour and relationship to the external world. The different clinical types include; Paranoid, disorganised/ hebephrenic, Catatonic, residual or latent, undifferentiated schizophrenia. The described patient is said to have a paranoid schizophrenia; it is characterised by having delusions ( an unrealistic belief) of persecution, reference, control. These delusions are usually well connected and also have no disturbance  of speech and motor behaviour.

A catatonic episode is characterised by a marked disturbance of motor behaviour.

 

Organic delirium is characterised by clouding of consciousness and disorientation.

136. A 40-year-old patient is registered in a narcological dispensary. Somatically: skin is dramatically hyperemic, sclera are injected, hyperhidrosis is present. BP- 140/100 mm Hg, heart rate - 100/min. Mental state: autopsychic orientation is intact, allopsychic orientation is distorted. The patient presents with motor anxiety. There is an expression of fear on his face. He refuses to talk about his problems and asks to release him immediately, because he ”may be killed”. This state developed in a day after one of his regular drinking bouts. What is your provisional diagnosis?

Explanation

Delirium tremens also known as Alcohol withdrawal delirium is the most severe form of alcohol withdrawal symptoms. It is characterised by an altered mental status and hyperactivity of the autonomic nervous system. Remember from the question stem, this started a day after one of his drinking bouts. 

Delirium tremens occurs in chronic alcohol abusers who abruptly discontinue alcohol use, often as early as 24 - 48 hours. Alcohol acts as a central nervous system depressant. It enhances the effect of inhibitory neurotransmitters while down-regulating excitatory neurotransmitters. Abrupt cessation of alcohol causes a decrease in the inhibitory actions of GABA neurotransmitter resulting in overactivity of the central nervous system. 

 

The overactivity seen with alcohol withdrawal manifests as altered mental status, hallucinations (“may be killed”), anxiety, seizures, tremors, psychomotor agitation, insomnia etc.

137. A 35-year-old female patient has gained 20 kg weight within a year with the normal diet. She complains of chill, sleepiness, dyspnea. The patient’s mother and sister are corpulent. Objectively: height - 160 cm, weight - 92 kg, BMI- 35,9. Obesity is uniform, there are no striae. The face is amimic. The skin is dry. The tongue is thickened. Heart sounds are muffled. Heart rate - 56/min., BP- 140/100 mm Hg. The patient has constipations, amenorrhea for 5 months. TSH- 28 mkME/l (normal rate - 0,32-5). Craniogram shows no pathology. What is the etiology of obesity in this case?

Explanation

Your thyroid produces thyroid hormone, which controls many activities in your body, including how fast you burn calories and how fast your heart beats. Diseases of the thyroid cause it to make either too much or too little of the hormone. Depending on how much or how little hormone your thyroid makes, you may often feel restless or tired, or you may lose or gain weight. Women are more likely than men to have thyroid diseases, especially right after pregnancy and after menopause.

Hypothyroidism is when your thyroid does not make enough thyroid hormones. It is also called underactive thyroid. This slows down many of your body's functions, like your metabolism. Signs and symptoms include the following: Feeling cold when other people do not; Constipation; weight gain, even though you are not eating more food; Feeling sad or depressed; Feeling very tired; Pale, dry skin; Dry, thinning hair; Slow heart rate (bradycardia); puffy face;  hoarse voice; abnormal menstrual bleeding etc.

Note that the craniogram shows no pathology so we can rule out Hypothalamic-pituitary. Hypo-ovarian can present with amenorrhea, vaginal dryness, monophasic cycles etc but not with bradycardia or weight gain. Absence of abdominal striaes or skin hyperpigmentation and sometimes diabetes - rules out the option of cushing’s disease (hypercorticoidism). 

 

The increased TSH levels seen in this patient implies an activation of the feedback mechanism signalling  the body to produce more thyroid hormones; T3 and T4.

138. A 10 week pregnant woman was admitted to a hospital for recurrent pain in the lower abdomen, bloody discharges from the genital tracts. The problems developed after a case of URTI. The woman was registered for antenatal care. Speculum examination revealed cyanosis of vaginal mucosa, clean cervix, open cervical canal discharging blood and blood clots; the lower pole of the gestational sac was visible. What tactics should be chosen?

Explanation

This is a case of retained product of conception which probably is an outcome of a missed abortion. Retained product of conception is a very common gynecological condition that you will definitely experience in your everyday Gynecological practice. She had acute respiratory viral infection (ARVI) which must have caused the abortion/miscarriage and presented with bloody discharge along with blood clots with no evidence of a live fetus, just the gestational sac was visible. In such conditions, a curettage or Manual Vacuum Aspiration (MVA) is done to get out the retained product of conception and the bleeding will stop.

Indication for curettage includes Abnormal uterine bleeding: irregular bleeding, menorrhagia, suspected malignant or premalignant condition,Retained material in the endometrial cavity, Evaluation of intracavitary findings from imaging procedures (abnormal endometrial appearance due to suspected polyps or fibroids),

 

Evaluation and removal of retained fluid from the endometrial cavity (hematometra, pyometra) in conjunction with evaluating the endometrial cavity and relieving cervical stenosis etc.

139. A patient with fibromyoma of uterus sized up to 8-9 weeks of pregnancy consulted a gynaecologist about acute pain in the lower abdomen. Examination revealed pronounced positive symptoms of peritoneal irritation, high leukocytosis. Vaginal examination revealed that the uterus was enlarged corresponding to 9 weeks of pregnancy due to the fibromatous nodes, one of which was mobile and extremely painful. Appendages were not palpable. There were moderate mucous discharges. What is the optimal treatment tactics?

Explanation

 

From vaginal examination, we observe the presence of a very large fibroid mass that causes the enlargement of the uterus and is responsible for the severe pain the woman is experiencing and the mucous discharge. Coupled with the positive symptom of peritoneal  irritation, this patient is in need of an urgent surgical procedure.

140. A 4-month-old child was admitted to a surgical department 8 hours after the first attack of anxiety. The attacks happen every 10 minutes and last for 2-3 minutes, vomiting occurred once. Objectively: the child’s condition is grave. Abdomen is soft, palpation reveals a tumor-like formation in the right iliac area. After rectal examination the doctor’s finger was stained with blood. What is the most probable diagnosis?

Explanation

Ileocaecal invagination is also termed ileocaecal intussusception which makes up more than 70% of cases of intussusception in children. Note that the tumour-like formation is in the right iliac area ( the ileocaecal angle - the point at which the ileum and caecum meet). Intussusception is a medical condition in which one section of the intestine gets enclosed or enveloped by another part. Symptoms include, vomiting, abdominal pain, and rectal bleeding (red jelly-like stools).

 

Wilms tumor is also called nephroblastoma and is a rare tumor that occurs in children, also presents with abdominal pain, anorexia, vomiting, malaise but does not show any signs of rectal bleeding.

141. A woman has focal encephalitis in the anamnesis. Her spatial orientation is not disrupted. She has a sensation, as if ”everything seems surreal: buildings are small, round or distorted; trees are upside down; people are very tall with thin limbs”. Determine the psychopathologic syndrome:

Explanation

 

Derealization and depersonalization are both types of dissociative disorders. In these cases, the person feels detached or separated from His or her own mental process. In derealization form, the individual feels detached from the environment 9 surrounding) while in depersonalization , the individual feels detached from his/her own life ( the feeling of an outsider). These conditions can be triggered by severe stress. Notice that this  patient observes psychological changes in the environment eg buildings, trees etc; this indicates a derealization syndrome. Oneiric syndrome is usually associated with dreams.

142. A 56-year-old woman complains of itching skin of her torso, constant nausea, constipation, sensation of heaviness and pain in the right subcostal area, extreme general fatigue. The patient suffers from biliary cirrhosis. The skin is pale icteric. The abdomen is soft, the liver protrudes 2,0 cm from under the margin of the right costal arch, sensitive on palpation. Biochemical investigation: total bilirubin - 142,0 mcmol/l, conjugated bilirubin - 139,0 mcmol/l, alanine aminotransferase - 0,98 mmol/hour·l, aspartate aminotransferase - 0,82 mmol/hour·l, alkaline phosphatase - 8,7 mmol/hour·l. What drug should be prescribed in the first place?

Explanation

143. A 40-year-old female patient complains of having a bulge on the anterior surface of neck for 5 years. Objectively: Ps- 72/min., arterial pressure - 110/70 mm Hg, in the right lobe of thyroid gland palpation reveals a mobile node 4x2 cm in size, the left lobe is not palpable, the basal metabolic rate is 6%. What is the most likely diagnosis?

Explanation

Notice that the Blood pressure and heart rate are normal which indicates no case of hyperthyroidism or hypothyroidism. Recall that A goiter occurring in a hyperthyroid gland is a Hyperthyroid goiter; one that occurs in a hypothyroid gland is a hypothyroid goiter while a goiter that occurs in a normal environment  is an Euthyroid goiter. This is a nodular form because on palpation, a mobile node was palpated.
144. Estimation of community health level involved analysis of a report on diseases registered among the population of district under charge (reporting form 12). What index is calculated based on this report?

Explanation

Common morbidity rate refers to the proportion or frequency at which a disease occurs in a population; from the report of registered diseases in a particular area, we can calculate the  morbidity rate.
145. A 32-year-old woman addressed a dermatologist with complaints of slightly itching rashes in the mouth angles. She has been suffering from this condition for 3 days. Objectively: there are isolated small phlyctenas and superficial erosions covered in honey-yellow scabs against the background of slight hyperemia. Make the diagnosis:

Explanation

 

The statement  ‘isolated small phlyctenas and superficial erosions covered in honey-yellow scabs’ clearly describes an impetigo which is the most common manifestation of streptoderma. Impetigo is characterized by a sudden onset. On the reddened background of the skin appears a bubble, the size of a pea, filled with muddy yellowish contents. This bubble very quickly grows in size, up to 1-2 cm, then it opens, exposing the erosive surface with scraps of the epidermis around the periphery. Almost instantly occurs drying of this element with the formation of honey-yellow crusts. This is accompanied by severe itching.

146. After a contact with chemicals a plant worker has suddenly developed stridor, voice hoarseness, barking cough, progressing dyspnea. Objective examination reveals acrocyanosis. What is your provisional diagnosis?

Explanation

 

The contact with chemicals led to the edematic reaction in the worker’s larynx. Recall that the larynx is the voicebox; this is why the edematic reaction presents with symptoms such as stridor, voice hoarseness, barking cough etc.

147. An 18-year-old woman complains of pains in her lower abdomen, purulent discharge from the vagina, temperature rise up to 37, 8oC. Anamnesis states that she had random sexual contact the day before the signs appeared. She was diagnosed with acute bilateral adnexitis. On additional examination: leukocytes in the all field of vision, bacteria, diplococci with intracellular and extracellular position. What agent is most likely in the given case?

Explanation

All of the above listed options can lead to an Adnexitis which refers to a disease of the female genital organs that causes inflammation of the uterus, ovaries and fallopian tubes. Note that on further examination, , diplococci with intracellular and extracellular position were found- this characteristic is typical for N. gonorrhea which is a gram negative diplococci.
148. A 45-year-old man has been suffering from duodenal ulcer disease for 5 years. He complains of weakness, dizziness, dryness of the skin. Objectively: the skin and visible mucosa are pale, chapped lips; heart rate is 100/min., BP- 100/70 mm Hg, systolic murmur at all points on heart auscultation. All other internal organs are unchanged. Fecal occult blood test is positive. Blood test: erythrocytes - 3, 1 · 1012/l, Hb- 88 g/l, color index - 0,7, leukocytes - 4, 6 · 109/l, platelets - 350·109/l, ESR- 21 mm/hour, anisocytosis, poikilocythemia, serum iron - 9,5 mcmol/l. What treatment tactics would you choose?

Explanation

This patient suffers from duodenal ulcers for a long period; a positive occult blood test indicates bleeding from the GI tract ( most likely from the ulcers). Notice that the blood values show signs of anemia ( normal erythrocyte count in Male: 4.3 − 5.9 · 1012/L); hemoglobin- Male: 135-175 g/L and color index 0.85- 1.05. A reduction in the color index and hemoglobin levels indicate an iron deficiency anemia; this can be confirmed by the presence of  RBCs of various sizes and shapes ( anisocytosis and poikilocytosis). The recommended iron content in blood ranges from 9-30 mmol/L. This patient should be given supplementary iron and diet changes should be made to meet up the body’s requirement.
149. A 58-year-old patient complains of a headache in the occipital region, nausea, choking, opplotentes. The presentations appeared after a physical exertion. Objectively: the patient is excited. Face is hyperemic. Skin is pale. Heart sounds are regular, the 2nd aortic sound is accentuated. BP- 240/120 mm Hg, HR- 92/min. Auscultation reveals some fine moist crackles in the lower lungs. Liver is not enlarged. ECG shows signs of hypertrophy and left ventricular overload. What is the most likely diagnosis?

Explanation

 

A person is said to be in a state of hypertensive crisis if the blood pressure exceeds 180mmhg for systolic and 110mmhg for diastolic pressure. The above patient presents with signs of ventricular hypertrophy and overload which complicates his current state of a hypertensive emergency. Other complications of hypertensive crisis include pulmonary edema, a stroke, aneurysm, heart failure etc.

150. A 45-year-old patient (14-year-long work record as a house painter) upon the contact with synthetic paint develops skin reddening, edema, severe itching and oozing lesions on her face. Symptoms disappear after the contact with this chemical substance stops but even the smell of paint alone is enough to make them reappear each time. Each recurrence is characterised by increased severity of symptoms. What provisional diagnosis can be made?

Explanation

 

Eczema (dermatitis ) is a chronic inflammatory disease of the skin characterized by the presence of red, itchy, dry scaly rashes. Occupational or professional  eczema (dermatitis) is gotten from contact with certain chemicals in the course of working (in this case due to contact with the chemicals in paint ). Allergic dermatitis is a form of contact dermatitis that results from immune reactions towards certain irritants.

151. A 32-year-old woman addressed a maternity clinic with complaints of infertility that has been lasting for 7 years. Her menstrual cycle occurs in two phases. Hysterosalpingography reveals obstruction of the uterine tubes in the ampullar areas, an adhesive process in the small pelvis can be observed. What treatment is most advisable in this case?

Explanation

Recall that fertilization takes place in the ampulla of the uterine cavity; from Hysterosalpingography ( a radiologic procedure used to examine the uterine cavity), we find that there is an obstruction in this area. In modern times, a laparoscopic procedure is the best type of surgery to be carried out; also known as a keyhole or minimal invasive type of surgery, it involves the use of a laparoscope ( a camera like device that allows the surgeon have a better view of the cavity in view).  On the other hand, a laparotomy or celiotomy involves a large incision and is an open type of surgery.
152. Survey radiograph of a 52-year-old worker of an agglomeration plant (28 years of experience, the concentration of metal dust is 22-37 mg/m3) shows mildly pronounced interstitial fibrosis with diffused contrast well defined small nodular shadows. The patient has no complaints. Pulmonary function is not compromised. What is the provisional diagnosis?

Explanation

Agglomeration plants are factories that work mainly with Iron ore materials together with other products, refining them into finished goods. This  patient has an accumulation of iron dust in his system as a result of long term exposure over the years. Pneumoconiosis is a restrictive lung disease  caused by the inhalation of dust leading to fibrosis. Depending on the type of dust inhaled, different types exist. They include; Siderosis (iron ore), byssinosis (cotton), anthracosis (coal), Asbestosis (asbestos), silicosis (silica dust).
153. A 5-year-old child has body temperature increased up to febrile numbers, suffers from inertness, weakness. Examination revealed hemorrhage on the skin of limbs and torso. Enlargement of cervical and axillary lymph nodes can be detected. The liver is 4 cm below the costal arch; the spleen is 6 cm below the costal arch. Blood test: erythrocytes - 2, 3 · 1012/l, Hb- 60 g/l, platelets - 40 · 109/l, leukocytes - 32, 8 · 109/l, eosinophiles - 1%, band neutrophiles - 1%, segmented neutrophiles - 12%, lymphocytes - 46%, monocytes - 1%, blasts - 40%, Duke’s bleeding time is 9 min. What examination is necessary to make the diagnosis?

Explanation

Notice that from the above analysis, the platelet level is relatively low (150 − 400 · 109/L is the normal). This is confirmed in the result of the duke test being 9 minutes - prolonged ( normal is 2-5 ). The  Duke test is done to check the bleeding time , it is a platelet function test. An abnormality indicates diseases such as von Willebrand, thrombocytopenia, DIC etc.  Recall that platelets originate from large bone marrow cells ( megakaryocytes)  and a bone marrow biopsy will be most helpful in analysis amongst all the above listed.  
154. A child from primipregnancy was born in a term labor and has body weight of 4000 g and body length of 57 cm. When born, he was nonresponsive to examination. Diagniosis is diffuse. Heart rate is 80/min. What resuscitation measures should be taken?

Explanation

 For resuscitation of the baby, the CAB approach should be used ( circulation, airway and breathing). From the question, we see that the baby’s circulation is present although reduced ( HR- 80 bpm; should be higher for babies); to assist the baby in breathing, An ALV ( assisted lung ventilation should be used). An intubation is only done if there is a restriction or obstruction in the upper airways or in cases.

 
155. A 15-year-old teenager has undergone medical examination in military recruitment center. The following was revealed: interval systolic murmur at the cardiac apex, accent of the II heart sound over the pulmonary artery, tachycardia. What additional examination method will be most informative for determining a diagnosis?

Explanation

An echocardiography is a procedure used in checking the live images of the heart. Information from this procedure shows: Changes in your heart size, Pumping strength, Damage to the heart muscle, Valve problems, Heart defects.

An electrocardiography is a procedure that  records electrical signals of the heart. It provides information about the heart rate, rhythm, Inadequate blood and oxygen supply to the heart, Heart attack and some structural abnormalities.

 

A phonocardiography is the recording of the sounds from the heart.

156. A 64-year-old patient has been hospitalised with complaints of progressive jaundice that developed over 3 weeks ago without pain syndrome, along with general weakness, loss of appetite. Objectively: temperature is 36, 8oC, heart rate is 78/min., abdomen is soft and painless, peritoneum irritation symptoms are not detected, palpation reveals sharply enlarged tense gallbladder. What disease can be characterised with these symptoms?

Explanation

 

Cancer of the head of the pancreas appears near the common bile duct. From an early stage, they tend to compress this duct leading to an obstruction in bile flow( causing an obstructive jaundice). Note that Cholecystitis usually presents with tenderness (peritoneal irritation) in the right upper quadrant and signs of systemic infection (pyrexia, raised leukocyte count, raised C-reactive protein). The above patient presents with none of these symptoms thereby ruling out the options of cholecystitis.

157. A 6-year-old girl attended a general practitioner with her mother. The child complains of burning pain and itching in her external genitalia. The girl was taking antibiotics the day before due to her suffering from acute bronchitis. On examination: external genitalia are swollen, hyperemic, there is white deposit accumulated in the folds. The most likely diagnosis is:

Explanation

Candidal vulvovaginitis is also known as vaginal thrush or vaginal yeast infection. It is characterised by very severe vaginal itching, burning sensation while urinating, pain during sex, hyperemic vagina and a thick white vaginal discharge. It occurs due to excessive growth of vaginal candida. Note that Trichomoniasis secretion is usually yellowish-green in colour.
158. A patient with otopyosis is in sharply deteriorating condition: he developed headache, vomiting, febrile temperature, general hyperesthesia. Meningeal symptoms and stagnant optic disks are observed. There are no focal symptoms. Liquor is turbid, blood pressure is high, albuminocytological dissociation occurs with neutrophils. What disease can be suspected?

Explanation

Recall that purulent meningitis occurs due to a bacterial ( meningococcus, pneumococcus etc) origin while serous meningitis has viral origin ( enterovirus etc). Primary purulent meningitis is mainly an independent disease ( has no underlying disease) while the secondary form is usually a complication of an underlying disease. Notice that this patient has Otopyosis ( a bacterial infection of the ear that results in purulent discharge). This infective process has likely spread to the meninges resulting in its inflammation and the above listed signs (meningeal symptoms, general hyperesthesia etc).
159. A 25-year-old patient is not married and has sexual relations with several partners. During the last 3 months he noticed small amount of mucoserous secretions produced from urethra. Subjectively: periodical itching or burning pain in urethra. Two months ago pain in the knee join developed. Possibility of trauma or exposure to cold is denied by the patient. During the last week eye discomfort is noted - lacrimation and itching. What provisional diagnosis can be suggested?

Explanation

 

Reactive arthritis is an autoimmune condition that develops in response to an infection in another part of the body. Coming into contact with bacteria and developing an infection can trigger reactive arthritis. It has symptoms similar to various other conditions collectively known as "arthritis,". It is caused by another infection and is thus "reactive". The symptoms of reactive arthritis very often include a combination of three seemingly unlinked symptoms—an inflammatory arthritis of large joints, inflammation of the eyes (conjunctivitis and uveitis), and urethritis. A useful mnemonic is "the patient can't see, can't pee and can't bend the knee" or "the patient can't see, can't pee and can't climb a tree". Also known as Reiter’s syndrome. 

160. . A 19-year-old patient complains of severe pain in axillary crease. Condition onset was a week ago after her swimming in a cold river and epilation. The next day painful ”boil” appeared that was becoming larger every day and became a plum-sized tumor. Upon examination nodular conical growths joined together are detected, the skin covering them is bluish-red in color. Some nodules have fistulous openings producing thick purulent mass. Body temperature is 38, 5oC, general malaise. The most likely diagnosis is:

Explanation

161. Annual report of an in-patient ward presents data about the number of patient days and the number of patients, who have undergone treatment within a year. What work indicator of the in-patient ward can be calculated based on these data?

Explanation

Average duration of in-patient treatment refers to the Average length of stay;  calculated by dividing the sum of inpatient days by the number of patients admissions with the same diagnosis-related group classification.

Hospital Bed Turnover Rate is the number of times there is change of occupant for a bed during a given time period

It is given by the formula:

Hospital Bed turnover rate = Number of discharges (including deaths) in a given time period / Number of beds in the hospital during that time period.

 

Bed occupancy rate is calculated by the no. of beds occupied (bed days) divided by the total no. of beds available  for a year, all multiplied by 100%.

162. ECG revealed the following in a 10- year-old child: sharp acceleration of the heart rate - 240/min., P wave overlaps with T wave and deforms it, moderate lengthening of PQ interval, QRS complex is without changes. What pathology does this child have?

Explanation

163. A 54-year-old patient complains of weakness, jaundice, itching skin. Disease onset was 1,5 months ago: fever up to 39oC appeared at first, with progressive jaundice developed 2 weeks later. On hospitalisation jaundice was severely progressed. Liver cannot be palpated. Gallbladder is enlarged and painless. Blood bilirubin is 190 micromole/l (accounting for direct bilirubin). Stool is acholic. What is the most likely jaundice genesis in this patient?

Explanation

Prehepatic/ hemolytic - occurs due to increased breakdown of RBC eg, hemolytic disease of newborn. There is an increased level of unconjugated bilirubin. Stool and urine colour are normal.

Hepatic/ parenchymatous  - Occurs in diseases affecting the liver parenchyma eg, cirrhosis, hepatitis etc. There is an increase in both unconjugated and conjugated bilirubin. Urine appears dark and faeces pale.

Post hepatic/ obstructive/ mechanical jaundice - pathology lies after conjugation of bilirubin and is caused by obstruction of biliary path. Conjugated bilirubin is accumulated, Urine is dark and faeces pale or acholic. Seen in disease such as cholelithiasis. 

Caroli syndrome is a rare genetic disease in which the common bile duct is unusually wide. It is characterised by bile duct stones, jaundice and flu like symptoms.
164. A 22-year-old patient complains of 8- month delay of menstruation. Anamnesis: menarche since the age of 12,5. Since the age of 18 menstruations are irregular. No pregnancies. Mammary glands have normal development; when the nipples are pressed, milk drops are discharged. On gynecological examination: moderate uterus hypoplasia. On hormonal examination: prolactin level exceeds the norm two times. On computed tomogram of the sellar region: a spaceoccupying lesion 4 mm in diameter is detected. The most likely diagnosis is:

Explanation

Notice that the result of the CT shows a bulky mass in the sella; anatomically, the pituitary gland is located in the sella turcica of the sphenoid bone and a bulky formation of such diameter in this location indicates the presence of a tumour. Furthermore, Prolactin levels are twice the normal; this hormone is produced from lactotrophs present in the anterior pituitary gland. The increased production is mostly as a result of the tumour.

 

Lactational amenorrhea is observed in breastfeeding mothers; the period after child birth in which the lactating mother doesn't menstruate.

165. A 2-year-old child in a satisfactory condition periodically presents with moderate proteinuria, microhematuria. US results: the left kidney is undetectable, the right one is enlarged, there are signs of double pyelocaliceal system. What study is required to specify the diagnosis?

Explanation

Excretory Urography is also known as intravenous Pyelogram. It is an x-ray procedure that involves the intravenous administration of contrast material inorder to  verify and localize upper urinary tract diseases.  Retrograde pyelogram also uses a contrast agent for better visualization of the ureters and kidneys but note that: In intravenous pyelogram, the contrast dye is injected into  a vein while in retrograde pyelogram, it is injected directly into the ureters. Retrograde pyelogram is mostly done in cases where excretory urography does not give a clear image of the pathological area.
166. A 13-year-old girl complains of fatigability, frequent headaches, cardialgia. Eight years ago she had a case of pyelonephritis. Urine analyses periodically revealed leukocyturia. The child has undergone no further treatment. On examination: increased BP up to 150/100 mm Hg. Ultrasound investigation revealed significant reduction of the right kidney. What process is leading in arterial hypertension pathogenesis in this case?

Explanation

 

From ultrasound, we observe the reduction in size of the right kidney; this may be due to a reduction in blood flow or a chronic infection ( as earlier stated, she previously had a case of pyelonephritis). Due to the constriction of the renal artery, the kidney releases Renin (from juxtaglomerular cells). This renin converts angiotensinogen (produced in the liver) to angiotensin I; Angiotensin I is then converted to angiotensin II by angiotensin converting enzyme, ACE ( an enzyme produced in the vascular endothelial cells of the lungs). Angiotensin II then stimulates the secretion of aldosterone from the adrenal cortex. In this system, angiotensin II is a potent vasoconstrictor that assists in the Increase of blood pressure.

167. A 26-year-old patient with affective bipolar disorder has developed a condition manifested by mood improvement, behavioural and sexual hyperactivity, verbosity, active body language, reduced need for sleep. Which of the following drugs are most efficient in this case?

Explanation

A neuroleptic is an antipsychotic drug used that targets a patient's cognition and behaviour.  In psychotic patients, neuroleptic drugs cause a reduction in confusion and agitation and tend to normalize psychomotor activity. This patient also has a reduced need for sleep therefore, he should also be given a neuroleptic that can assist in sleeping ( sedative effect). Tranquilizers are used in anxiety disorders, fear, tension etc. Nootropics are drugs used in boosting memory and creativity.
168. A 32-year-old man complains of pain in the chest on the left, dyspnea, temperature rise up to 38, 0oC, slight cough. The disease onset was 2 weeks ago after overexposure to cold. He had suffered from bronchoadenitis in his childhood. The affected side lags during breathing; percussion reveals dull sound with oblique margin in the lower left lung, where breathing is absent. The right heart border is displaced outwards. Mantoux test with 2 TU resulted in a papule 16 mm in size. What diagnosis is most likely?

Explanation

169. An emergency doctor has diagnosed a 32-year-old woman with generalized convulsive status epilepticus. The deterioration in the patient’s condition is caused by a sudden gap in the epilepsy treatment. Specify the doctor’s further tactics:

Explanation

Status epilepticus is a medical emergency in which a person experiences a seizure that lasts more than five minutes or consecutive seizures between five minutes. Benzodiazepines and phenytoin are drugs for the first line of treatment.
170. A 19-year-old patient complains of dyspnea during physical exertion. He often has bronchitis and pneumonia. Cardiac murmur has been observed since his childhood. On auscultation: there is splitting of the II heart sound over pulmonary arteria, systolic murmur in the third intercostal space near the left sternum margin. ECG test shows right bundle-branch block. What is the provisional diagnosis?

Explanation

Atrial Septal Defect: A mid-systolic, Crescendo-decrescendo murmur, loudest at the upper left sternal border with a ‘fixed split second heart sound’.

N/B S2 split is normal during inspiration but a fixed split means both during inspiration and expiration.

Patent Ductus Arteriosus: Normal S1 with a continuous Crescendo-decrescendo ‘Machinery’ murmur ( machine gun like) or  A systolodiastolic murmur in the II intercostal space on the left. This murmur may continue during the second heart sound, making the 2nd second heart sound difficult to hear.

N/B small sized PDA presents with no abnormal sound.

Aortic Stenosis: ( most common valvular disease). Causes: Idiopathic, Rheumatic heart disease. 

Characteristic Murmur: Heard at aortic area (2nd intercostal space, right sternal border) Ejection-systolic, high pitched murmur with ‘ Crescendo-decrescendo’ character. Radiates to the carotids, often presents with slow rising pulse and narrow pulse pressure. Patients may complain of exertional syncope

N/B Aortic stenosis causes Left ventricular hypertrophy

Mitral Regurgitation: Causes: Age related, Ischemic Heart disease, infective endocarditis, Rheumatic diseases,connective tissue disorders eg. Marfan’s syndrome, Ehler Danlos’ syndrome. 

Characteristic Murmur: Heard at Mitral area ( 5th intercostal space, midclavicular line), A Pansystolic, high pitched, ‘whistling’ murmur’ which radiates to the left axilla. A 3rd heart sound might be heard.

N/B Mitral Regurgitation causes left atrial dilatation

 
171. A 75-year-old male patient complains of slight pain in the right iliac region. The abdominal pain arose 6 days ago and was accompanied by nausea. Surgical examination revealed moist tongue, Ps- 76/min., BP- 130/80 mm Hg. Abdomen was soft, slightly painful in the right iliac region on deep palpation, the symptoms of the peritoneum irritation were doubtful. In blood: RBCs- 4, 0·1012/l, Hb- 135g/l, WBCs- 9, 5 · 109/l, stab neutrophils - 5%, segmentonuclear - 52%, lymphocytes - 38%, monocytes - 5%, ESR- 20 mm/hour. Specify the doctor’s further tactics:

Explanation

172. An 8-year-old boy has a 2-year history of blotchy itchy rash appearing after eating citrus fruit. The first eruption occurred at the age of 6 months after the introduction of juices to the baby’s diet. Father has a history of bronchial asthma, mother - that of allergic rhinitis. What is the most likely diagnosis?

Explanation

Atopic dermatitis is a systemic chronic allergic disease that occurs in people with an inherited predisposition to atopy and characterized by typical morphological changes of the skin with itching, lesions of the central and autonomic nervous system, endocrine and immune systems with hyper Ig E.    Essential criteria for diagnosis: itch, rash elements`, typical morphology and localization ( flexor surface of extremities in adults (lichenification), extensor surfaces and face in children (eczema)), chronic recurrent course, atopic diseases in personal and family anamnesis (bronchial asthma, allergic rhinitis).

Contact dermatitis: a localized reaction that includes redness, itching, and burning where the skin has come into contact with an irritant such as an acid, a cleaning agent, or other chemical.

Allergic contact eczema (dermatitis): a red, itchy, weepy reaction where the skin has come into contact with a substance that the immune system recognizes as foreign.
173. A 25-year-old patient has been admitted to the hospital with the following problems: weakness, sweating, itching, weight loss, enlarged submandibular, cervical, axillary, inguinal lymph nodes. Objectively: hepatomegaly. Lymph node biopsy revealed giant Berezovsky-Reed-Sternberg cells, polymorphocellular granuloma composed of lymphocytes, reticular cells, neutrophils, eosinophils, fibrous tissue, plasma cells. What is the most likely diagnosis?

Explanation

Berezovsky/Reed-Sternberg (RS) cells are giant cells that have more than one nucleus and appear large, eosinophilic, inclusion-like nuclei. The lineage of these cells is probably B-cell lymphocytes. RS cells and the accompanying mononuclear Hodgkin’s cells are the neoplastic cells in Hodgkin’s lymphoma (lymphogranulomatous), surrounded by a reactive cellular infiltrate. RS cells usually express CD15 and CD30 (Ki-1, an antigen that is also expressed in anaplastic large cell lymphoma).
174. An 8-year-old child was hospitalized for fever up to 39, 8oC, inertness, moderate headache, vomiting. Examination revealed meningeal symptoms. Lumbar puncture was performed. The obtained fluid had raised opening pressure, it was transparent, with the cell count of 450 cells per 1 mcl (mainly lymphocytes - 90%), glucose level of 2,6 mmol/l. What causative agent might have caused the disease in the child?

Explanation

 

Enterovirus infections (ECНO and Coxsackie’s infections) a group of an acute diseases caused by ECНO and Coxsackie’s enteroviruses, that  are characterized by the variety of clinical displays from the mild fever and simple carrying of virus to protracted meningoencephalitis, myocarditis, myalgia and other. Notice that the cell count in the CSF is 90% lymphocytes which indicates a viral cause;  for bacterial origin, the cell count will be mainly neutrophils. Enterovirus is the only viral cause amongst the listed agents.

175. A 25-year-old patient during selfexamination detected a tumor in the upper external quadrant of the right mammary gland. On palpation: painless, dense, mobile growth 2 cm in diameter is detected in the mammary gland; no changes in the peripheral lymph nodes are observed. On mammary glands US: in the upper external quadrant of the right mammary gland there is a spaceoccupying lesion of increased echogenicity 21х18 mm in size. The most likely diagnosis is:

Explanation

An adenoma is a benign epithelial tumor from the epithelium of the glands and glandular organs e.g. breast, thyroid gland, ovaries etc. A fibroadenoma is a benign nodular proliferation and not a true neoplasm (cancer); it presents as a mobile lump in the breast of young women. Lactocele are breast cysts that develop during or after lactation and are characterised by the accumulation of milk.
176. 2 weeks after labour a parturient woman developed breast pain being observed for 3 days. Examination revealed body temperature at the rate of 39oC, chills, weakness, hyperaemia, enlargement, pain and deformity of the mammary gland. On palpation the infiltrate was found to have an area of softening and fluctuation. What is the most likely diagnosis?

Explanation

Mastitis is inflammation of the breast tissue and can be broken down into lactational and non-lactational mastitis. Lactational mastitis is the most common form of mastitis. Lactational mastitis, also known as puerperal mastitis, is typically due to prolonged engorgement of milk ducts, with infectious components from the entry of bacteria through skin breaks. Patients can develop a focal area of erythema, pain, and swelling, and can have associated systemic symptoms, including fever. This occurs most commonly in the first six weeks of breastfeeding but can occur at any time during lactation, with most cases falling off after 3 months. Lactational mastitis is most commonly caused by bacteria that colonize the skin, with Staphylococcus aureus being the most common. Risk factors for lactational mastitis include prior history of mastitis, nipple cracks and fissures, inadequate milk drainage, maternal stress, lack of sleep, tight-fitting bras, and use of antifungal nipple creams.

From the question stem, we can differentiate the different types of mastitis listed. On palpation, an INFILTRATE was found with an area of softening and fluctuation (PURULENT). We can boldly conclude it is purulent because lactational/puerperal mastitis is commonly caused by bacteria which will produce a purulent inflammation.

177. A 48-year-old woman has been suffering from chronic pancreatitis for the last 7 years. Lately she has been noticing an increase in daily feces with foul smell, abdominal distention, gurgling. The patient complains of diarrhea, weakness, fatigability, loss of appetite, loss of weight. What syndrome can be suspected in this case?

Explanation

Malabsorption syndrome is observed when the small intestine is unable to absorb certain nutrients and fluid into the bloodstream. Causes include trauma, surgery, infection, deficiency diseases, underlying diseases such as pancreatitis, cystic fibrosis etc.  This patient has a history of chronic pancreatitis; recall that the pancreas produces enzymes that help in digestion ie., trypsin, chymotrypsin, lipase, amylase. In the absence of these enzymes, certain food materials will not be broken down properly thus, absorption will be disturbed.

 

Irritable colon or irritable bowel syndrome is accompanied by symptoms such as cramping abdominal pain, diarrhea, constipation, bloating and gas etc

178. A 37-year-old man suffers from attacks of unconsciousness, dyspnea during physical exertion, periodical sensations of heart rate disorder. Father of the patient died suddenly at the age of 45. Objectively: heart rate is 90/min., BP is 140/90 mm Hg. On heart US: ejection fraction - 49%, significant myocardium thickening of the left ventricle and interventricular septum. What drug should be prescribed for the treatment?

Explanation

 Note that The patient above presents with no kidney related issue thereby ruling out the option of Enalapril ( an ACE inhibitor), furosemide (loop diuretic) and hydrochlorothiazide (thiazide diuretic) - these drugs will be the first to be considered in a case of hypertension coupled with a kidney related  issue. Bisoprolol ( beta blocker) is the best possible choice, It is a beta 1 selective blocker and acts by competitively blocking adrenaline’s stimulation of beta 1 adrenergic receptors causing a reduction of heart rate and increase in contractility.

 

Also take note of the ejection fraction ( 55-70 % is the normal)- this patient’s ejection fraction is low which indicates a reduction in the heart’s pumping ability ( contractility); as earlier stated, beta 1 blockers increases heart contractility. 

179. During a regular check-up of a 50-yearold woman a tumor was detected in her right mammary gland. The tumor is 5 cm in diameter, dense, without clear margins. The skin over the tumor resembles lemon rind, the nipple is inverted. The lymph node can be palpated in the axillary region. What diagnosis is most likely?

Explanation

From physical examination, there is a dense tumor ‘without clear margins’ in the right mammary gland - this description is typical for breast cancer. Signs of breast cancer include; presence of lumps in the breast, change in size and shape of the breast, skin depression in the affected area, swollen lymph nodes etc.  

A breast lipoma is a non-cancerous tumor (benign) tumor composed of adipose ( fatty) tissues with a thin fibrous capsule around it. These types of tumors have clear margins. 

 

A lacteal cyst is also called galactocele; usually seen shortly after lactation and is characterised the presence of a milk filled  cavity

180. A 20-year-old woman complains of sensation of air shortage, lingering dull pain in the heart area, irritability. Objectively: general condition is satisfactory, heart rate lability, BP is 130/60 mm Hg. ECG reveals disruption of repolarization proceses. The patient is diagnosed with somatoform autonomic dysfunction of cardial type. Specify the conditions of the patient treatment:

Explanation

Note that this patient has a somatoform autonomic dysfunction- which is more a mental disorder than physical. The symptoms are presented by the patient as if they were due to a physical disorder of a system or organ that is largely or completely under autonomic innervation and control, i.e. the cardiovascular, gastrointestinal, respiratory, and  urogenital systems. Physically, nothing is wrong with the patient. This patient should get an outpatient treatment.

 

Recall that in outpatient care, the individual does not need to be admitted into the hospital whereas inpatient care requires admission.

181. A 38-year-old man’s workplace is within the area of effect of ionizing radiation. During regular medical check-up he expresses no complaints. Blood test: erythrocytes - 4, 5 · 1012/l, Нb- 80 g/l, leukocytes - 2, 8 · 109/l, platelets - 30 · 109/l. Can this person continue to work with sources of ionizing radiation?

Explanation

 

Observe that the levels of WBC, hemoglobin and platelets are very low. This can be traced to the effect of radiation from around his workplace. Working in this environment should be prohibited for this individual because of the health effects.

182. A maternity patient breastfeeding for 1,5 weeks has attended a doctor. She considers the onset of her disease to be when proportional breast engorgement occurred. Mammary glands are painful. Body temperature is 36, 6oC. Expression of breast milk is hindered. The most likely diagnosis is:

Explanation

They key finding here is “ the expression of breastmilk is hindered” - this refers to a stagnation in the flow of milk in one or several lobes of the mammary gland leading to an enlarged and painful mammary gland; if not corrected, this leads to mastitis ( is an inflammation (most often one-sided) of the mammary gland caused by pathogenic coccal flora (mostly staphylococci)).
183. An 11-year-old boy complains of general weakness, fever up to 38, 2oC, pain and swelling of the knee joints, sensation of irregular heartbeat. 3 weeks ago, the child had a case of tonsillitis. Knee joints are swollen, the overlying skin and skin of the knee region is reddened, local temperature is increased, movements are limited. Heart sounds are muffled, extrasystole is present, auscultation reveals apical systolic murmur that is not conducted to the left inguinal region. ESR is 38 mm/hour. CRP is 2+, antistreptolysin O titre - 400. What is the most likely diagnosis?

Explanation

Acute rheumatic fever typically occurs 2-3 weeks after Group A streptococcal pharyngitis. Diagnosis depends on a set of clinical signs that result from this infection ( Jones criteria)- these include chorea, carditis, subcutaneous nodules, erythema marginatum, and migratory polyarthritis. For biochemical analysis, certain key markers are used in diagnosis; these include C reactive protein ( an inflammatory marker), antistreptolysin O titer ( a marker that indicates a group A streptococcal infection ), ESR ( a marker from general blood analysis). Using these criterias, we can observe that this patient has arthritis, carditis and results from the biochemical analysis all appear positive for an acute rheumatic fever.

 

Reactive arthritis is also known as reiter’s syndrome  and refers to the inflammation of joints as a result of  a urogenital, intestinal or respiratory infection.

184. An 18-year-old woman complains of mammary glands swelling, headaches, tearfulness, abdominal distension occurring the day before menstruation. The symptoms disappear with the beginning of menstruation. Menstruations are regular, last for 5-6 days with interval of 28 days in between. Gynecological examination revealed no changes of internal genitals. What is your diagnosis?

Explanation

Premenstrual syndrome is a wide variety of signs and symptoms that affects a woman’s emotion, physical health, and behavior during certain days of the menstrual cycle, generally just before her menses. Symptoms start five to 11 days before menstruation and typically go away once menstruation begins. Signs include abdominal pain and bloating, vomiting ,meteorism, change in sleep patterns, emotional instability etc.

Also called Stein- Leventhal syndrome, Ovarian sclerocytosis is the process of ovarian regeneration, accompanied by the formation of small cystic formations up to 1 cm in size. It usually occurs in patients with polycystic ovarian syndrome. Key findings include; infertility, male pattern hair distribution, weight loss, hormonal disbalance, bilateral enlargement of the ovaries, violation of menstrual cycle etc. 

 

Sheehan’s syndrome is one of the major causes of hypopituitarism in females; it is due to pituitary infarction as a result of postpartum hemorrhage.  fatigability, significant weight loss, weakness, and loss of appetite all include associated symptoms.

185. A 24-year-old patient had been delivered to a thoracic department with a chest injury, a fracture of the IV, V, VI ribs on the right. Plan radiography shows the fluid level in the pleural cavity reaching the III rib on the right. Puncture blood contained clots. What is the optimal treatment tactics?

Explanation

The above patient presents with a major trauma to the right side of the chest which is confirmed by a collapsed right lung on x ray. Thoracotomy is a surgical procedure carried out inorder to have access to thoracic organs such as the lungs, heart or esophagus. This procedure is the first step in gaining access to the thoracic cavity and thus is indicated in cases of a lobectomy, major trauma of the chest, pneumonectomy. In the case of this patient, an urgent thoracotomy should be carried out in order to take out the blood clots. thoracocentesis is indicated in pleural effusion and is also referred to as a pleural puncture.
186. A 30-year-old man was delivered to a neurosurgical department with complaints of constant headaches, nausea, vomiting, fever, weakness of the right-side limbs. Anamnesis states that one month ago the patient had a surgery for left-sided suppurative otitis and mastoiditis. He has been undergoing treatment in an ENT department. Approximately 2 weeks ago the temperature increased, and the patient developed headaches. Objectively: heart rate - 98/min., BP- 140/90 mm Hg, temperature - 38, 3oC. Neurologically pronounced stiff neck: bilateral Kernig’s symptom, unsteadiness during the Romberg’s maneuver. Computer tomography of the brain revealed a threedimensional growth with a capsule in the left hemisphere. Make the diagnosis:

Explanation

Recall that most infections of the paranasal sinuses and mastoid process spread to the surrounding structures eg brain if not treated on time and properly. From Anamnesis, this patient had a surgical procedure carried out for the issue of suppurative otitis media and mastoiditis. The positive kernig’s sign and the result from the romberg’s maneuver indicates a neurological dysfunction most likely from the spread of the initially stated pathology ( otitis and mastoiditis). A brain abscess refers to a pus filled cavity in the brain; the result from the CT confirms this diagnosis.

 

In echinococcus, a cerebral hydatid cyst is usually found with a history of contact with infected dogs.

187. Examination of a Rh-negative pregnant woman at 32 weeks of gestation revealed a four-time rise of Rh-antibody titer within 2 weeks, the titer was 1:64. In the first two pregnancies the patient had experienced antenatal fetal death due to hemolytic disease. What is the optimal tactics of pregnancy management?

Explanation

 

Rh Antibody Titre test is done to determine the type and quantity of antibodies in the blood. Rh antibodies are likely to rise during pregnancy. Depending on the level of other antibodies these antibodies could cause hemolytic problems in the baby and need to be monitored. It is usually repeated several times during pregnancy (at 32, 36, & 38 weeks). A low titer (less than 1:16) may not pose any problem for the baby.  any test from 1:64 or higher, is indicative of incompatibility. Notice that the patient has previously had 2 fetal deaths from this incompatibility, to save the current situation, an early delivery should be induced.

188. A 74-year-old patient visited a urologist with complaints of pain above the pubis and inability to urinate for 8 hours. At home he had taken antispasmodics and had a warm bath but no improvement occurred. Objectively: the abdomen is soft and painful above the pubis; dullness of percussion sound is observed above the pubis. Murphy’s (Pasternatski’s) punch sign is negative on the both sides. What condition does the patient have?

Explanation

Acute Urinary Retention is a Sudden inability to urinate. The patient experiences increasingly agonizing suprapubic pain associated with severe urgency and may dribble (release) only small amounts of urine. Urinary retention is characterised by poor urinary stream with intermittent flow, straining, a sense of incomplete voiding, and hesitancy (a delay between trying to urinate and the flow actually beginning). As the bladder remains full, it may lead to incontinence, nocturia (need to urinate at night), and high frequency. Notice that this patient has only had this for 8 hours (a short period); in the chronic form, it might last for days and also be recurrent.

 

paradoxal Ischuria is a form urinary incontinence that occurs when the bladder is so full that it continually leaks urine; often attributable to a blocked urethra (e.g., due to prostate enlargement) or weak bladder muscles or nerve damage.

189. A 7-year-old child became ill again 2 weeks after a case of tonsillitis. There are the following complaints: temperature rise up to 38oC, hemorrhagic rash on the extremities, enlargement of the ankle joints. Blood test: hemoglobin is 120 g/l, platelets are 170 · 109/l, ESR is 30 mm/hour. Urine test: proteinuria up to 0,7 g/l, cylinders - 5-6 in the field of vision, erythrocytes - 8-10 in the field of vision. What mechanism of hemorrhagic syndrome is present in the given case?

Explanation

From the description above, the patient is said to have an Immune complex Vasculitis. This is a form of vasculitis, which is characterized by the deposition of immunoglobulin and/or complement on the vessel wall. It affects mainly small vessels and the kidneys; we can confirm this by the presence of hemorrhagic rash and signs of a kidney related disorder ( proteinuria, erythrocytes and cylinders in blood).

 

Observe that the platelet levels is normal ( norm. 150-400 · 109/l); this rules out the option of a platelet dysfunction.

190. A 33-year-old patient has developed dyspnea during physical exertion, palpitations, disruptions of heart rate, swollen legs. In the childhood the patient had a case of acute rheumatic fever that required in-patient treatment. There were no further requests for medical care. Objectively: heart rate is 92/min., rhythmic; BP is 110/70 mm Hg. At the apex the I heart sound is increased, triple rhythm, diastolic murmur. What heart disease is most likely?

Explanation

191. A 64-year-old woman has been suffering from diabetes mellitus for the last 14 years. Approximately 3 days ago the skin on the distal phalanx of the I toe on the left foot became cold and bluish-black in color. Mild pain is observed in the affected area. Pulse on the pedal arteries cannot be detected, pulse on the popliteal artery is retained. Glycemia is 12,4 mmol/l. US scan: stenosis of the shin arteries, collateral compensated blood flow. Ankle-brachial pressure index is 0,7. Foot Xray: destruction of the distal phalanx of the I toe. Determine the grade of diabetic foot according to Wagner:

Explanation

192. A 4-month-old boy has been undergoing in-patient treatment for pneumocystic pneumonia for 4 weeks. The diagnosis has been made based on clinical signs, typical X-ray presentation, presence of severe hypoxemia, positive dynamics caused by intravenous introduction of Biseptol (Cotrimoxazole). Anamnesis states that enzymelinked immuno sorbent assay (ELISA) detected antibodies to HIV in the umbilical blood. Polymerase chain reaction (PCR) was performed on the child at the ages of 1 month and 3 months, and proviral DNA was detected in the child’s blood. Viral load and number of CD4+-lymphocytes was not measured. Make the diagnosis:

Explanation

The Results from the ELISA test shows the presence of HIV antibodies in the umbilical blood while PCR shows Proviral DNA in Child’s blood; These both indicate that the child had been infected by HIV in Utero. HIV is a retroviral infection. a single-stranded, positive-sense, enveloped RNA virus which contains virally encoded enzymes such as Reverse transcriptase, integrase, protease etc A reduction in CD4 positive cells is a hallmark for this disease.
193. A 35-year-old patient undergoing treatment for heart disorder in cardiological department has developed complaints of acute sudden pain in the epigastrium, temperature rise up to 38, 3oC. Blumberg’s and Razdolsky’s (abduction of femur) signs are positive. What necessitates surgical aid in the given case?

Explanation

194. A 5-year-old girl was hospitalized with complaints of pain and swelling in the right knee joint, temperature rise up to 38, 4oC and a rash diagnosed as erythema annulare centrifugum. The signs developed 3 days after the recovery from a case of acute respiratory disease. Name the etiotropic drug to be prescribed:

Explanation

 

From anamnesis, The patient had a respiratory infection 3 days prior and now presents with a skin lesion most likely caused by an infection. An etiological treatment is one which is directed at the causative agent ( in this case the microorganism). Augmentin is an antibacterial combination ( amoxiclav) which consists of Amoxicillin and clavulanic acid ( beta lactamase inhibitor). Diclofenac sodium will be used for symptomatic treatment since its effects are on pain and inflammatory reduction.

195. During medical examination of a group of children under 4 years carried out by a pediatric team in one of the African countries a set of similar pathological signs was detected in some of the children. The signs are as follows: growth inhibition, mental changes, muscle atrophy, swellings, changes in hair and skin pigmentation. These children were diagnosed with kwashiorkor. What food products should be added to the diet to treat this disorder?

Explanation

 

Kwashiorkor is a disease marked by severe protein malnutrition and bilateral extremity swelling. It usually affects infants and children. It should be differentiated from marasmus which is energy or calorie deficiency. For the correction of kwashiorkor, foods rich in protein should be taken eg, fish, meat, cereals, milk etc.

196. An 8-year-old girl was diagnosed with signs of Morgagni-Adams-Stokes disease that developed against the background of the III degree atrioventricular heart block. What drug should be introduced intravenously for emergency aid?

Explanation

Atropine is an Anticholinergic drug, it is a competitive antagonist of the muscarinic acetylcholine receptor types M1, M2, M3, M4 and M5. About its effect on the heart, atropine is used in treatment of bradycardia,  second-degree heart block Mobitz type 1 (Wenckebach block), and also third-degree heart block with a high purkinje or AV-nodal escape rhythm. It is usually not effective in second-degree heart block Mobitz type 2, and in third-degree heart block with a low Purkinje or ventricular escape rhythm.

Dobutamine is used in treatment of cardiogenic shock and severe heart failure; Digoxin is a cardiac glycoside used for atrial fibrillation, atrial flutter, and heart failure.
197. A 62-year-old man addresed a urologist with complaints of frequent urination at night (5-6 times per night), sensation of incomplete voiding of the urinary bladder, pain in the lower abdomen, slow urination. Anamnesis: the II degree essential hypertension (peak BP is 160/100 mm Hg). Current case: the II degree enlargement of the prostate gland, PSA is 2,2 ng/ml. Select the drug suitable for long-term therapy of the patient’s combined pathology:

Explanation

Doxazosin is an alpha-1 adrenergic blocker used in the treatment of high blood pressure and an enlarged prostate ( both of which are seen in the above patient). All the other options ie., propranolol ( beta blocker), Indapamide ( thiazide diuretic), Amlodipine ( potassium channel blocker) and captopril (ACE inhibitor) are all indicated in hypertension but not in hypertrophy of the prostate.
198. On the 9th day after childbirth the obstetric patient developed high fever up to 38oC. She complains of pain in the right mammary gland. The examination revealed the following: a sharply painful infiltrate can be palpated in the right mammary gland, the skin over the infiltrate is red, subareolar area and nipple are swollen and painful. What is your diagnosis?

Explanation

A breast abscess is a localised collection of pus in the breast tissue. It is usually caused by a bacterial infection. It is characterised by the presence of a lump on palpation (painful infiltrate can be palpated), swollen and painful nipple. In mastopathy, we observe changes such as swellings, nodules, cysts etc, it is hormone dependent. In cancer, there will be increased proliferation of atypical tissues.

 
199. A 16-year-old girl addressed a doctor with complaints of fatigability and dizziness. On heart auscultation: systolic murmur in the II intercostal area along the breastbone edge on the left. ECG revealed signs of the right ventricular hypertrophy. X-ray revealed dilatation of the the pulmonary artery trunk, enlargement of the right heart. What heart disorder is it?

Explanation

Dilatation refers to the heart muscles being stretched and thinned thereby increasing the capacity of the affected chamber but reducing its pumping ability. Hypertrophy is the Thickening of the heart muscle due to prolonged or increased stress on the heart. The major reason for the dilatation of the pulmonary artery and enlargement of the right heart is due to a major resistance that reduces or stops the outflow of blood from the pulmonary artery; A pulmonary stenosis will be the major cause of this changes in the pulmonary artery; other cause may include a pulmonary valve regurgitation.   

Note: Ejection-systolic murmurs can also be heard in Pulmonary valve stenosis (2nd intercostal space, left sternal border) and Hypertrophic Obstructive Cardiomyopathy in children (4th intercostal space on the left sternal border). 

200. A 32-year-old patient complains of excessive appetite, excess weight, dyspnea during physical exertion. There are fat deposits in the area of abdomen and shoulder girdle. The skin is pale-pink, adult male pattern of hair distribution is observed on the torso, no stretch marks. Heart rate is 90/min., BP is 120/80 mm Hg, body build index equals 35. Blood sugar is 4,9 mmol/l, cholesterol is 6,2 mmol/l. On ophthalmoscopy: fundus of the eye without changes. What provisional diagnosis can be made?

Explanation

Android type obesity can be differentiated from the gynoid form by the location of fat storage. In the android form, fat is stored in the abdominal region while in the gynoid form, fats are stored around the thighs and hip area. This patient has the android form because his fats distribution is around the abdominal region. 

 

Recall that the hypothalamus controls our rate of eating and drinking; a secondary hypothalamic obesity will be seen in a patient with a hypothalamic tumour or lesion. In neuroendocrine obesity, hormones that control fats breakdown and metabolism especially those produced from the pituitary gland are disturbed.